To Revise Flashcards

1
Q

CURB 65 CRITERIA

A
Confusion
Urea >7mM
RR >30
BP <90
>_65
How well did you know this?
1
Not at all
2
3
4
5
Perfectly
2
Q

Complications of pneumonia

A
Lung abscess
Empyema
Sepsis
AF
Pleural effusion
Resp failure 
Hypotension
How well did you know this?
1
Not at all
2
3
4
5
Perfectly
3
Q

Empyema- presentation, ix, rx

A

Presentation - recovering from pneumonia becomes feverish again
Ix - TAP - pH <7.2, decrease glucose, high LDH
Mx - US guided chest drain + abx

How well did you know this?
1
Not at all
2
3
4
5
Perfectly
4
Q

Mx of any condition

A

A-E if acute

Look at all symptoms and see how to treat each one individually

How well did you know this?
1
Not at all
2
3
4
5
Perfectly
5
Q

SIRS Criteria

A
at least 2 of 
Temp >38 or <36 
HR >90
RR >20
WCC > 12x10^9
How well did you know this?
1
Not at all
2
3
4
5
Perfectly
6
Q

Causes of non resolving pneumonia

A
CHAOS
Complication 
Host
Abx
Organism
Secondary diagnosis
How well did you know this?
1
Not at all
2
3
4
5
Perfectly
7
Q

Causes of transudate effusions

A

Common

  • HF
  • hypoalbuminaemia
  • cirrhosis

Less common

  • hypothyroidism
  • PE
  • Mitral stenosis

Rare

  • SVCO
  • constrictive pericarditis
How well did you know this?
1
Not at all
2
3
4
5
Perfectly
8
Q

Causes of exudative pleural effusions

A

Common
Malignancy - particularly gynae
Infection - pneumonia, TB, legionella

Less common
Inflammation - RA, pancreatitis, dressler’s
Lymphatic drainage issues

How well did you know this?
1
Not at all
2
3
4
5
Perfectly
9
Q

Lights criteria - when to use and what is it

A

When protein in pleural aspirate is between 25 and 30

Criteria
Exudate if one or more of the following:

Pleural fluid/serum protein >0.5
Pleural fluid/serum LDH >0.6
Pleural fluid LDH > 2/3 of upper limit of normal

How well did you know this?
1
Not at all
2
3
4
5
Perfectly
10
Q

Ix to diagnose pleural effusion

A

US guided pleural aspiration

CXR

How well did you know this?
1
Not at all
2
3
4
5
Perfectly
11
Q

Pulmonary effusion CXR

A

Meniscus

Blunted costaphrenic angles

How well did you know this?
1
Not at all
2
3
4
5
Perfectly
12
Q

Tension pneumothorax mx

A

A-E
Insert large bore IV cannula into 2nd ICS MC line
Chest drain into affected side

How well did you know this?
1
Not at all
2
3
4
5
Perfectly
13
Q

Causes of pnuemothorax

A

Spontaneous
primary - without disease - young thin men, smokers
secondary - with disease eg COPD, Marfan’s, Ehlers Danlos, ppulmonary fibrosis, sarcoidosis

Traumatic

Iatrogenic

  • Central line insertion
  • positive pressure ventilation
  • pacemaker insertion
How well did you know this?
1
Not at all
2
3
4
5
Perfectly
14
Q

Pneumothorax rf

A
Height
Male
COPD, marfans, ehlers danlos, sarcoidosis 
Procedure eg central line or pacemaker 
Smoker - particularly of cannabis 
Trauma
Diving
How well did you know this?
1
Not at all
2
3
4
5
Perfectly
15
Q

Rf of PE

A

SPASMODICAL

Sex: f
Pregnancy
Age: older
Surgery
Malignancy
Oestrogen
DVT
Immobility
Colossally obese
Antiphospholipid antibodies 
Lupus anti-coagulant
How well did you know this?
1
Not at all
2
3
4
5
Perfectly
16
Q

PE - from front door to treament

A

A-E
Wells Score to determine probability
If highly probable - CTPA
If low probability - D Dimer. If D Dimer high - CTPA. If low - exclude PE
Give interim anticoagulation - rivoroxaban or apixaban first line. If CI, give LMWH

Treatment
Pharmaco - oral anticoags eg warfarin, DOACs (in UHL), LMWH, fondaparinux
Mechanical - IVC filter
Thrombolytic - if large. Streptokinase

How well did you know this?
1
Not at all
2
3
4
5
Perfectly
17
Q

Wells Score Categories

A

Symptoms of DVT
Previous VTE
Immobilisation for 3 days or surgery in the past 4 weeks
HR >100
Cancer
Haemoptysis
An alternative diagnosis is less likely than PE

If >4points PE likely

How well did you know this?
1
Not at all
2
3
4
5
Perfectly
18
Q

PE Ix

A
CTPA
Doppler calfs
Clotting factors
ECG
CXR
ABG
How well did you know this?
1
Not at all
2
3
4
5
Perfectly
19
Q

4 causes of ILD they want us to know about

A

Extrinsic allergic alveolitis
Usual Interstitial pneumonia (UIP)
Non specific interstitial pneumonia (NSIP)
Sarcoidosis

How well did you know this?
1
Not at all
2
3
4
5
Perfectly
20
Q

Signs of ILD

A

Fine inspiratory respiratory crackles

Clubbing

How well did you know this?
1
Not at all
2
3
4
5
Perfectly
21
Q

Symptoms of Extrinsic allergic alveolitis (acute and chronic)

A

Acute 4-8hrs after exposure

  • dyspnoea
  • dry cough
  • fever, rigors, malaise

Chronic

  • increasing dyspnoea
  • weight loss
  • T1 resp failure
  • cor pulmonale
How well did you know this?
1
Not at all
2
3
4
5
Perfectly
22
Q

Sarcoidosis definition

A

Multistystem granulomatous disease of unknown origin

How well did you know this?
1
Not at all
2
3
4
5
Perfectly
23
Q

Symptoms of sarcoidosis

A

GRANULOMA
General - fever, malaise, lymphadenopathy
Respiratory - 90% have dry cough, dyspnoea, chest pain, reduced lung function
Arthralgia
Neurological - Bells palsy, meningitis, SOL
Urinary - increased calcium - renal stones
Low hormones - pituitary - amenorrhoea
Opthalmological - uveitis, sjrogens
Myocardial - restrictive cardiomyopathy secondary to granulomas, pericardial effusion
Abdominal - splenomegaly and hepatomegaly

How well did you know this?
1
Not at all
2
3
4
5
Perfectly
24
Q

CXR finding of sarcoidosis

A

BHL

How well did you know this?
1
Not at all
2
3
4
5
Perfectly
25
Q

Ix of sarcoidosis

A
PFTs
ECHO and ECG for cardiac innvolvement
CT head for neuro involvement
Urinary calcium levels 
CXR
How well did you know this?
1
Not at all
2
3
4
5
Perfectly
26
Q

Treatment principles of ILD

A
A to E
Remove exposure
Stop smoking
Treatment of infective exacerbations
MDT approach
Palliation
Transplantation
How well did you know this?
1
Not at all
2
3
4
5
Perfectly
27
Q

Ddx for resp issues that is apt to think about

A

covid19

How well did you know this?
1
Not at all
2
3
4
5
Perfectly
28
Q

Types of Lung Cancer

A

Small cell

Non small cell - SCC, Adeno, Large

How well did you know this?
1
Not at all
2
3
4
5
Perfectly
29
Q

Differences in presentation between SCLC and NSCLC (SCC and adeno)

A

SCLC

  • smokers
  • centrally located near bronchioles
  • typically presents as advanced metastatic disease
  • associated with cushings

SCC

  • smokers
  • centrally located
  • slow to metastasise
  • locally invasive
  • PTHrP secreting

Adeno

  • non-smokers
  • peripherally located
  • often presents with metastases
How well did you know this?
1
Not at all
2
3
4
5
Perfectly
30
Q

Group symptoms/signs of cancers

A

Local/organ
General
Metastatic

How well did you know this?
1
Not at all
2
3
4
5
Perfectly
31
Q

How to group complications of cancers

A

Local
Paraneoplastic
Metastatic

How well did you know this?
1
Not at all
2
3
4
5
Perfectly
32
Q

Signs of lung cancer

A

Lungs - consolidation, collapse, pleural effusion
General - cachexia, clubbing, anaemia, lymphadenopathy
Metastatic - bone pain, hepatomegaly, confusion fits focal neurological signs

How well did you know this?
1
Not at all
2
3
4
5
Perfectly
33
Q

NSCLS Mx

A

MDT
Surgical resection - lobectomy, pulmonectomy, wedge resection + adjuvant chemo
Chemo
Chemo + radio

How well did you know this?
1
Not at all
2
3
4
5
Perfectly
34
Q

SCLC Mx

A

MDT
Can do radio but tend to relapse
Palliation and analgesia
Very poor prognosis

How well did you know this?
1
Not at all
2
3
4
5
Perfectly
35
Q

Complications of lung cancer

A

Local

  • SVCO
  • RLN palsy
  • phrenic nerve palsy
  • Horner’s syndrome (pancoast cancer)

Paraneoplastic

  • hypercalcaemia
  • SIADH
  • Cushings
  • Acanthosis Nigricans

Metastatic

  • pathological fractures
  • hepatomegaly
  • addisons
  • confusion, fits, focal neurology
How well did you know this?
1
Not at all
2
3
4
5
Perfectly
36
Q

Presentation of TB

A

Fever with drenching night sweats
Haemoptysis
Weight loss
Productive cough with purulent sputum

How well did you know this?
1
Not at all
2
3
4
5
Perfectly
37
Q

TB risk factord

A
  • TB contact
  • travelled to a high risk TB country or born there
  • immunocompromised
How well did you know this?
1
Not at all
2
3
4
5
Perfectly
38
Q

Anti - TB therapy plus side effects and initial ix

A

Rifampicin - hepatitis and rashes, can cause orange secretions
Isoniazid - hepatitis and rashes, peripheral neuropathy
Pyrazinamide - hepatitis and rashes, arthralgia
Ethambutol - retrobulbar neuritis

LFTs need close monitoring
Visual acuity assessed before ethambutol
Pyridoxine given whilst on isoniazid as prophylaxis against peripheral neuropathy

RIPE for 2 months followed by RI for 4 months

How well did you know this?
1
Not at all
2
3
4
5
Perfectly
39
Q

Mx principles of TB

A

negative pressure side room

ziel nielsson stain of sputum

How well did you know this?
1
Not at all
2
3
4
5
Perfectly
40
Q

Causes of bronchiectasis

A

Post infective

  • whooping cough
  • TB

Immune deficiency
- hypogammaglobulinanaemia

Genetic Mucocilliary Clearance issues

  • CF
  • Kartageners syndrome

Secondary Immune Deficiency

  • RA
  • HIV

Toxic Insult
- gastric aspiration

Also Allergic Bronchopulmonary Aspergillosus

How well did you know this?
1
Not at all
2
3
4
5
Perfectly
41
Q

Ix of bronchiectasis

A
Immunoglobulin levels
Rheumatoid factor
Cystic fibrosis gene screening
Aspergillus IgE
Auto antibodies

Definitive = High Contrast CT

How well did you know this?
1
Not at all
2
3
4
5
Perfectly
42
Q

Definition of a bronchiectasis exacerbation

A

A patient with bronchiectasis who presents with a deterioration of 3 or more of the following over 48hrs

  • cough
  • sputum volume and consistency
  • Sputum purulence
  • Breathlessness
  • fatigue
  • haemoptysis
How well did you know this?
1
Not at all
2
3
4
5
Perfectly
43
Q

What are pts with bronchiectasis at increased risk of

A

Recurrent infections

How well did you know this?
1
Not at all
2
3
4
5
Perfectly
44
Q

What conditions is ABPA associated with

A

asthma
bronchiectasis
CF

How well did you know this?
1
Not at all
2
3
4
5
Perfectly
45
Q

Ix of ABPA

A

Raised aspergillus IgE as well as total IgE

How well did you know this?
1
Not at all
2
3
4
5
Perfectly
46
Q

What type of pathogen is aspergillus fumigatus

A

Fungi

How well did you know this?
1
Not at all
2
3
4
5
Perfectly
47
Q

Mx of ABPA

A

Steroids

How well did you know this?
1
Not at all
2
3
4
5
Perfectly
48
Q

How is Cf diagnosed

A

Newborn screening
Sweat chloride test
CF genotyping

How well did you know this?
1
Not at all
2
3
4
5
Perfectly
49
Q

How does CF often intially present

A

Meconium ileus in newborns
Intestinal Malabsorption
Recurrent chest infections
Most picked up by newborn screening

How well did you know this?
1
Not at all
2
3
4
5
Perfectly
50
Q

Common CF complications

A
Recurrent chest infections
Malabsorption
Distal Intestinal Obstruction Syndrome
Infertility
CF related diabetes
How well did you know this?
1
Not at all
2
3
4
5
Perfectly
51
Q

Mx of CF

A
Lifestyle factors which I know
Chest physio
Mucolytics
Pancreatic enzyme replacement 
Long term antibiotics
ADEK replacement and nutritional support
Long term monitoring of CF diabetes
How well did you know this?
1
Not at all
2
3
4
5
Perfectly
52
Q

Pathophysiology of asthma

A

IgE release
Eosinophils
Inflammation
Increased goblet cells so increased mucus

How well did you know this?
1
Not at all
2
3
4
5
Perfectly
53
Q

Symptoms of mild asthma

A

PEFR >75

no symptoms of severe asthma

How well did you know this?
1
Not at all
2
3
4
5
Perfectly
54
Q

Symptoms of moderate asthma attack

A

PEFR 50-75%

No symptoms of severe asthma

How well did you know this?
1
Not at all
2
3
4
5
Perfectly
55
Q

Symtpoms of severe asthma

A
PEFR 33-50%
O2 < 92%
Cant complete full sentences 
Increased RR >25
HR >110
How well did you know this?
1
Not at all
2
3
4
5
Perfectly
56
Q

Symptoms of life threatening asthma attack

A

PEFR <33
Normal pCO2
Cyanotic, hypotensive, confused, increased respiratory effort, near or full silent chest, exhaustion, arrythmias

How well did you know this?
1
Not at all
2
3
4
5
Perfectly
57
Q

Near fatal asthma attack

A

Increased pCO2

How well did you know this?
1
Not at all
2
3
4
5
Perfectly
58
Q

Mx of asthma

A

A-E approach
Give O2 if less than 92%

5mg Neb Salbutamol
40mg oral prednisolone

If severe:
Neb 500 micrograms ipatropium bromide
Back to back salbutamol every 15 mins

If life threatening or near fatal
IV aminophylline
IV salbutamol
Urgent ITU or anaesthetist review

How well did you know this?
1
Not at all
2
3
4
5
Perfectly
59
Q

What drugs can trigger asthma

A

Aspirin and betblockers

How well did you know this?
1
Not at all
2
3
4
5
Perfectly
60
Q

COPD pathophysiology

A

Chronic bronchitis and emphysema
hyperplasia of mucous glands
Loss of cilial function
Chronic inflammation and fibrosis of small airways

How well did you know this?
1
Not at all
2
3
4
5
Perfectly
61
Q

Obstructive spirometry

A

FEV1:FVC = <0.7

Inwards ‘L’ shape spirometry curve - cant insert pic hence this is just description

How well did you know this?
1
Not at all
2
3
4
5
Perfectly
62
Q

3 main causes of COPD

A
  • smoking
  • occupational exposure/pollution
  • alpha 1 antitrypsin disorder
How well did you know this?
1
Not at all
2
3
4
5
Perfectly
63
Q

how to mx an acute exacerbation of COPD

A
  • A-E
  • O2 levels between 88 and 92
  • Nebs - salbutamol and ipatropium
  • Steroids - prednisolone 30mg stat and then for 7 days
    x
  • Abx if infective cause -
  • IV aminophylline
  • consider NIV if acidotic and type 2 resp failure
How well did you know this?
1
Not at all
2
3
4
5
Perfectly
64
Q

Talk about LTOT

A

specific criteria eg must be non smoker, O2 must be safe in house
has to be used for at least 16hrs a day for survival benefit
lose independence and reduced activity levels

How well did you know this?
1
Not at all
2
3
4
5
Perfectly
65
Q

aims of pulmonary rehabilitation

A

break the cycle of exercise and breathlessness. Avoid exercise that makes you breathless increasingly makes you more breathless

How well did you know this?
1
Not at all
2
3
4
5
Perfectly
66
Q

Causes of Type 1 resp failure (CO2 <6kPa)

A

V/Q mismatch

  • pneumothorax
  • early asthma
  • PE
  • PHT

Diffusion failure

  • Fluid (pulmonary oedema, pneumonia, infarction, blood)
  • Fibrosis
How well did you know this?
1
Not at all
2
3
4
5
Perfectly
67
Q

Causes of Type 2 Resp Failure (paCO2>6kPa)

A

V/Q mistmatch
Alveolar hypoventilation
- Obstructive - asthma, COPD, bronchiectasis epiglottitis
- Restrictive - CNS sedation, cervical cord lesion, fluid and fibrosis

How well did you know this?
1
Not at all
2
3
4
5
Perfectly
68
Q

Sx and signs of hypercapnia

A
  • headaches
  • peripheral vasodilation
  • confusion - coma
  • flap
  • bounding pulse
How well did you know this?
1
Not at all
2
3
4
5
Perfectly
69
Q

A-a gradient and normal value

A

Normal = <4
Greater than 4 means something wrong with lungs
PAO2 - PaO2

PAO2 = PIO2 - (PaCO2/0.8)

How well did you know this?
1
Not at all
2
3
4
5
Perfectly
70
Q

Causes of ARDS

A

Pulmonary

  • pneumonia
  • aspiration
  • inhalation injury

Systemic

  • sepsis
  • pancreatitis
  • DIC
How well did you know this?
1
Not at all
2
3
4
5
Perfectly
71
Q

Scale used to assess sleepiness in OSA

A

Epworth sleepiness scale

How well did you know this?
1
Not at all
2
3
4
5
Perfectly
72
Q

Mx of OSA

A

Weight loss
Sleep decubitus rather than supine
Mandibular advancement devices
CPAP - opens collapsed airways and improves V/Q mismatching
Should notify DVLA
Can use BiPAP if severe OSA with CO2 retention

How well did you know this?
1
Not at all
2
3
4
5
Perfectly
73
Q

Ix of Infective endocarditis

A

3 blood cultures at least an hr apart - gold
FBC, CRP, ESR, U+E(septic emboli)
Urine dip - haematuria - septic emboli
ECHO
ECG - long PR interval - perivalvular abcess

How well did you know this?
1
Not at all
2
3
4
5
Perfectly
74
Q

Most common IE organisms and people at risk of these

A

Strep viridans - Dental practice or long prosthetic
Staph aureus - IVDU
Staph epidermidis - short prosthetic

How well did you know this?
1
Not at all
2
3
4
5
Perfectly
75
Q

Abx for IE

A

Strep - benpen
Staph - fluclox

Pen allergic - vancomycin

How well did you know this?
1
Not at all
2
3
4
5
Perfectly
76
Q

Monitoring of IE

A

2 ECGs a week
1 ECHO a week
2 bloods a week
6 weeks of abx

How well did you know this?
1
Not at all
2
3
4
5
Perfectly
77
Q

Sign of fast AF

A

non palpable pulse bc loss of diastolic filling

How well did you know this?
1
Not at all
2
3
4
5
Perfectly
78
Q

Mx of persistent AF

A

Rate control
1st - beta blocker
2nd - diltiazem or verapamil

How well did you know this?
1
Not at all
2
3
4
5
Perfectly
79
Q

Mx of paroxysmal AF

A

Rhythm control

1st line - amiodarone

How well did you know this?
1
Not at all
2
3
4
5
Perfectly
80
Q

Amiodarone long term ADR

A
  • pulmonary fibrosis (BANSMe)

- hypothyroidism

How well did you know this?
1
Not at all
2
3
4
5
Perfectly
81
Q

When do you use warfarin and when use DOAC for AF? (Given the chadvasc score warrants its use)

A

Warfarin for valvular AF

DOAC for non-valvular

How well did you know this?
1
Not at all
2
3
4
5
Perfectly
82
Q

Mx of following situation
<48hr hx of acute AF
haemodynamically unstable

A

A-E

Emergency cardioversion - amiodarone or flecanide

How well did you know this?
1
Not at all
2
3
4
5
Perfectly
83
Q

Drugs used for different tachycardias

A

SVT - adenosine
VT - amiodarone
AF - amiodarone

How well did you know this?
1
Not at all
2
3
4
5
Perfectly
84
Q

Process of looking through ECG for tachycardias

A
Rate
Rhythm
Sinus or not
V1 - LBBB or RBBB, VT
Concordance
How well did you know this?
1
Not at all
2
3
4
5
Perfectly
85
Q

2 types of SVT

A

AVNRT

ANRT - only one we need to know is WPW. Seen by short PR and delta wave

How well did you know this?
1
Not at all
2
3
4
5
Perfectly
86
Q

Features of VT on ECG

A
Concordance
Broad QRS complex
AV dissociation - p waves all over the place
Capture and fusion beats
Regular

If irregular likely to be AF w/ BBB or pre excited AF

How well did you know this?
1
Not at all
2
3
4
5
Perfectly
87
Q

Causes of VT

A

IM QVICK

Iatrogenic - digoxin, anti arrhythmics
Myocarditis
QT increased
Valvular issues
Infarction
Cardiomyopathy
K low
K
How well did you know this?
1
Not at all
2
3
4
5
Perfectly
88
Q

Treatment for torsades de pointes

A

Magnesium sulfate

How well did you know this?
1
Not at all
2
3
4
5
Perfectly
89
Q

Mobitz 1 vs Mobitz 2

A

1 is increasing PR interval followed by dropped QRS

2 is same PR interval followed by dropped QRS

How well did you know this?
1
Not at all
2
3
4
5
Perfectly
90
Q

Causes of bradycardias

A

DIVISIONS

Drugs 
- Anti arrythmics (type 1a - amiodarone)
- Beta blockers
- Calcium channel blockers
- Digoxin
Ischaemia/inferior infarct
Vagal hypertonia
Infections
Sick sinus
Infiltration
O - hypOthermia, hypOthyroidism, hypOkalaemia 
Neuro - increased ICP
Surgery or septal defect
How well did you know this?
1
Not at all
2
3
4
5
Perfectly
91
Q

Hr <40 treatment

A

500 micrograms IV atropine

How well did you know this?
1
Not at all
2
3
4
5
Perfectly
92
Q

htn stages

A

1 >140/90
2 > 160/100
3 >180/110

How well did you know this?
1
Not at all
2
3
4
5
Perfectly
93
Q

HTN targets

A

<80 yrs <140/90

>80yrs <150/90

How well did you know this?
1
Not at all
2
3
4
5
Perfectly
94
Q

Differential of paroxysmal hypertension associated with headaches and sweating

A

Phaechromocytoma

How well did you know this?
1
Not at all
2
3
4
5
Perfectly
95
Q

What is a hypertensive crisis

A

an increase in BP which if sustained over a few hrs will cause end organ damage (LV failure, encephalopathy, aortic dissection, renal failure, unstable angina)

How well did you know this?
1
Not at all
2
3
4
5
Perfectly
96
Q

Mx of a hypertensive crisis

A

A-E

IV:
1 sodium nitroprusside - needs arterial line BP monitoring 
2 GTN
3 Labetalol
4 esmolol
How well did you know this?
1
Not at all
2
3
4
5
Perfectly
97
Q

Heart murmurs sounds

A

Aortic stenosis - luuuub dub
Aortic regurge - lub tahh
Mitral regurge - durrr durr
Mitral stenosis - lub durr

How well did you know this?
1
Not at all
2
3
4
5
Perfectly
98
Q

Mx of oesophageal ca

A

Most have mets on diagnosis so majority are palliative - median survival is 4mo after dx
Some can go for an ivor lewis oesophagectomy - prognosis 5% survive 5years

How well did you know this?
1
Not at all
2
3
4
5
Perfectly
99
Q

Achalasia pathophys

A

Degeneration of myenteric plexus
reduces peristalsis
LOS fails to relax

How well did you know this?
1
Not at all
2
3
4
5
Perfectly
100
Q

What di 3-5% of pts with achalasia go on to develop

A

Oesophageal SCC

How well did you know this?
1
Not at all
2
3
4
5
Perfectly
101
Q

Ix for achalasia and sign shown

A

Barium swallow - bird bea sign

OGD to exclude malignancy

How well did you know this?
1
Not at all
2
3
4
5
Perfectly
102
Q

How does the dysphagia in achalasia present

A

liquids then solids

How well did you know this?
1
Not at all
2
3
4
5
Perfectly
103
Q

H Pylori Triple therapy

A

PPI + Clarithromycin + Amoxicillin/metronidazole

How well did you know this?
1
Not at all
2
3
4
5
Perfectly
104
Q

Where are gastric and duodenal ulcers most likely found

A

Gastric - lesser curve if stomach antrum

Duodenal - 1st part of duodenum

How well did you know this?
1
Not at all
2
3
4
5
Perfectly
105
Q

What is the rockall score used for

A

Predict risk of re bleeding in upper GI bleeds

How well did you know this?
1
Not at all
2
3
4
5
Perfectly
106
Q

Symptoms and signs of gastric cancer

A

Sx

  • usually presents late
  • epigastric pain
  • fevers, w/l
  • dyspepsia
  • dysphagia

Signs

  • epigastric mass
  • anaemia
  • virchows node
  • sister mary josephs node
  • ascitis
  • acanthosis nigricans
How well did you know this?
1
Not at all
2
3
4
5
Perfectly
107
Q

Mx of gastric cancer

A

Mostly palliative due to late presentation
Medical - analgesia, PPI
Surgical - pyloric stenting or bypass

Curative surgical - endoscopic resection, partial or total gastrectomy

How well did you know this?
1
Not at all
2
3
4
5
Perfectly
108
Q

What is Zollinger Ellison and presentation

A

gastrin secreting tumour (gastrinoma)

Dyspepsia with chronic diarrhoea (bc of inactivation of pancreatic enzymes)

How well did you know this?
1
Not at all
2
3
4
5
Perfectly
109
Q

When to refer someone with GORD

A
If:
>65
Failed to respond to medical treatment
Haematemesis/malena
Weight loss
Anaemia
Progressive symptoms
Dysphagia
How well did you know this?
1
Not at all
2
3
4
5
Perfectly
110
Q

Surgery for GORD

A

Nissen fundoplication

How well did you know this?
1
Not at all
2
3
4
5
Perfectly
111
Q

Pathophys of Barrett’s oesophagus

A

Trauma to oesophagus

metaplasia -> dysplasia -> adenocarcinoma

How well did you know this?
1
Not at all
2
3
4
5
Perfectly
112
Q

Two types of hiatus hernia and which should you always treat even if asymptomatic

A

Rolling (15%) and sliding (80%) and mixed (5%)

Rolling should always be treated as risk of strangulation

How well did you know this?
1
Not at all
2
3
4
5
Perfectly
113
Q

Difference between rolling and sliding hiatus hernias

A

Rolling - gastro oesophageal junction remains in abdomen and part of stomach rolls out in tk the chest
Sliding - gastro oesophageal junction herniates into chest

How well did you know this?
1
Not at all
2
3
4
5
Perfectly
114
Q

Haematemesis Differential Acromym

A
VINTAGE
Vascular - varices
Inflammatory - PUD
Neoplasia - oesophageal or gastric
Trauma - Mallory Weiss, Boehaaves syndrome
Angiodysplasia and HHT
Generalised bleeding disorders - warfarin, thrombolytics
Epistaxis
How well did you know this?
1
Not at all
2
3
4
5
Perfectly
115
Q

Rectal Bleeding Differentials Acronym

A

DRIPING Arse

Diverticular disease
Rectal haemorrhoids 
Infection - shigella, E coli, campylobacter 
Polyps
Inflammation - IBD
Neoplasms
Gastric upper GI bleeding 
Anal fissure/ Angiodysplasia + HHT+ ischaemic colitis 
rse
How well did you know this?
1
Not at all
2
3
4
5
Perfectly
116
Q

5 stages of fracture healing

A

Haematoma formation
Inflammation
Proliferation - of osteoblasts and fibroblasts
Consolidation - woven bone to lamellar bone
Remodelling

How well did you know this?
1
Not at all
2
3
4
5
Perfectly
117
Q

What factros can extend healing time of fractures

A

Adult
Smoker
Diaphysial
Open

How well did you know this?
1
Not at all
2
3
4
5
Perfectly
118
Q

3 key principles of fracture mx

A

Reduce
Hold
Rehabilitate

How well did you know this?
1
Not at all
2
3
4
5
Perfectly
119
Q

Benefits of reduction

A
  • Tamponade bleeding
  • reduce inflammation
  • reduced neuropraxia risk
  • reduced ischaemia risk
How well did you know this?
1
Not at all
2
3
4
5
Perfectly
120
Q

classification of open fractures

A
Gustillo-Anderson
1 - <1cm, clean
2 - 1-10cm clean
3A >10cm, adequate skin coverage
3B >10cm, inadequate skin coverage
3C - any open fracture with vascular injury
How well did you know this?
1
Not at all
2
3
4
5
Perfectly
121
Q

Mx of open fractures

A
6A’s
Analgesia 
Anti sepsis - copious fluid irrigation, debridement, cover with betadine soaked bandages
Assess - NV status, soft tissues
Anti-tetanus - give vaccine
Abx - broad spec eg co amox 
Align - align and splint
How well did you know this?
1
Not at all
2
3
4
5
Perfectly
122
Q

Complications of fractures generally

A

Immediate

  • bleeding
  • neurovascular damage

Short term

  • compartment syndrome
  • infection
  • fat embolism

Long term

  • malunion
  • post traumatic OA
  • growth disturbance
  • complex pain syndromes
How well did you know this?
1
Not at all
2
3
4
5
Perfectly
123
Q

Seddon classification of nerve injuries

A

Neuropraxia - temp loss of conduction with loss of axon continuity
Axonotmeses - disruption of nerve axon. Get wallerian degeneration. recovery is possible
Neurotmesis - disruption of entire nerve fibre. Surgery needed, recovery not usually complete

How well did you know this?
1
Not at all
2
3
4
5
Perfectly
124
Q

What palsy can an anterior shoulder dislocation cause

A

axillary nerve

How well did you know this?
1
Not at all
2
3
4
5
Perfectly
125
Q

what palsy can a fracture of humeral shaft cause? deformity?

A

radial nerve - waiters tip

How well did you know this?
1
Not at all
2
3
4
5
Perfectly
126
Q

what palsy can an elbow dislocation cause? deformity?

A

Ulnar nerve - claw hand

How well did you know this?
1
Not at all
2
3
4
5
Perfectly
127
Q

what palsy can a hip dislocation cause? deformity?

A

sciatic nerve - foot drop

How well did you know this?
1
Not at all
2
3
4
5
Perfectly
128
Q

what palsy can a fracture of neck of fibula cause? deformity?

A

Fibular nerve - foot drop

How well did you know this?
1
Not at all
2
3
4
5
Perfectly
129
Q

common organisms causing septic arthritis

A

Neisseria gonnorhoea
Staph aureus
Strep pyogenes

How well did you know this?
1
Not at all
2
3
4
5
Perfectly
130
Q

ddx of septic arthritis

A

OA flare
haemarthritis
reactive arthritis
lymes disease

How well did you know this?
1
Not at all
2
3
4
5
Perfectly
131
Q

Ix of septic arthritis

A

Synovial fluid aspirate prior to abx - look for leucocytes, gram stain, microscopy and culture
2 x blood cultures at different times
Routine Bloods
X ray

How well did you know this?
1
Not at all
2
3
4
5
Perfectly
132
Q

Length of abx course for septic arthritis

A

2 weeks of IV then 2-4 weeks of oral

How well did you know this?
1
Not at all
2
3
4
5
Perfectly
133
Q

Complications of septic arthritis

A

OA

Osteomyelitis

How well did you know this?
1
Not at all
2
3
4
5
Perfectly
134
Q

Mx of compartment syndrome

A

Fasciotomy
Analgesia
Remove tight bandages, splints, casts
Monitor renal function for signs of rhabdo - high CK, or for reperfusion injury

How well did you know this?
1
Not at all
2
3
4
5
Perfectly
135
Q

Grade Classification used for OA

A

Kellgreen and Lawrence

How well did you know this?
1
Not at all
2
3
4
5
Perfectly
136
Q

What is a segond fracture? what is it pathognomonic of?

A

avulsion of lateral proximal tibial. ACL

How well did you know this?
1
Not at all
2
3
4
5
Perfectly
137
Q

difference in surgical methods for outer third and inner third of meniscus

A

outer third often sutured as good blood supply

inner third often trimmed

How well did you know this?
1
Not at all
2
3
4
5
Perfectly
138
Q

types of meniscal tears

A

longitudinal - bucket handle
Transverse - parrot beak
Degenerative
Vertical

How well did you know this?
1
Not at all
2
3
4
5
Perfectly
139
Q

Complications of knee arthroscopy

A
DVT
Damage to:
- Saphenous vein and nerve
- popliteal vessels
- perineal nerve
How well did you know this?
1
Not at all
2
3
4
5
Perfectly
140
Q

Complications of MCL injury

A

Saphenous nerve damage

Joint instability

How well did you know this?
1
Not at all
2
3
4
5
Perfectly
141
Q

What is the ITB

A

aponeurosis of the tensor fascia lata and the gluteus maximus

How well did you know this?
1
Not at all
2
3
4
5
Perfectly
142
Q

Rf of ITB syndrome

A

Repetitive flexion and extension eg runners

Bowleggedness

How well did you know this?
1
Not at all
2
3
4
5
Perfectly
143
Q

Special tests for ITB syndrome

A

Nobles - lie supine, finger on lateral femoral epicondyle. pt extends leg, pain at 30 degrees
Renne - pt stands, finger on lateral femoral epicondyle, pt squats, pain at 30 degrees as this is when ITB crosses over

How well did you know this?
1
Not at all
2
3
4
5
Perfectly
144
Q

Surgical mx for ITB syndrome

A

ITB release - but only if symptoms and loss of function for greater than 6 months

How well did you know this?
1
Not at all
2
3
4
5
Perfectly
145
Q

What condition to be aware of when looking at patella for a fracture

A

bipartite patella - failure of medial and lateral facets to fuse so held together by fibrocartilage

How well did you know this?
1
Not at all
2
3
4
5
Perfectly
146
Q

Mx of patella fracture

A

Conservative

  • ensure early extension
  • wear brace or cylinder cast

Surgical.
- ORIF with tension band wiring

How well did you know this?
1
Not at all
2
3
4
5
Perfectly
147
Q

How does ORIF with tension band wiring work for patella fractures

A

converts tensile force applied to patella during extension into a compression force

How well did you know this?
1
Not at all
2
3
4
5
Perfectly
148
Q

Complications of patella fractures

A

Reduced ROM

post traumatic OA at the patellofemoral joint

How well did you know this?
1
Not at all
2
3
4
5
Perfectly
149
Q

Which tibial plateau is injured more frequently

A

lateral due to varus force

How well did you know this?
1
Not at all
2
3
4
5
Perfectly
150
Q

Which nerve can be damaged in a tibial plateau fracture

A

common fibular nerve

How well did you know this?
1
Not at all
2
3
4
5
Perfectly
151
Q

What will you see on xray of tibial plateau fracture? what other scan is usually required?

A

Lipohaemarthrosis

CT

How well did you know this?
1
Not at all
2
3
4
5
Perfectly
152
Q

The presence of fat in a joint indicates what

A

an intraarticular fracture

How well did you know this?
1
Not at all
2
3
4
5
Perfectly
153
Q

Classification of tibial plateau fractures

A

Schatzker

How well did you know this?
1
Not at all
2
3
4
5
Perfectly
154
Q

Criteria to receive conservative mx for a tibial plateau fracture . What is the mx

A

uncomplicated

  • articular step <2mm
  • no ligament injury
  • no tibial subluxation

Non or partial weight bearing in a high knee brace for 8-12 weeks
Physio
Analgesia

How well did you know this?
1
Not at all
2
3
4
5
Perfectly
155
Q

Criteria to receive surgical mx for tibial plateau fracture and what is it

A

Complicated fracture

  • articular slope >2mm
  • tibial subluxation
  • ligament injuries

Compartment syndrome
Open fracture

ORIF - metaphyseal gap filled in with bone graft or substitutes

How well did you know this?
1
Not at all
2
3
4
5
Perfectly
156
Q

When may ORIF be unsuitable and what would you do instead

A

highly comminuted fractures/polytrauma
significant soft tissue injury

Do external fixation

How well did you know this?
1
Not at all
2
3
4
5
Perfectly
157
Q

why is risk of compartment syndrome and open fractures greater along the tibial shaft

A

Bc there is a lack of significant soft tissue envelope

How well did you know this?
1
Not at all
2
3
4
5
Perfectly
158
Q

Mx of tibial shaft fracture

A
Realign stat 
Monitor for compartment syndrome
Above Knee back-slab
If stable - sarmiento cast
If unstable - surgery - intramedullary nailing or ORIF with locking plates
Fibula fractures tend to be left alone
How well did you know this?
1
Not at all
2
3
4
5
Perfectly
159
Q

Causes of sensironeural hearing loss

A
acoustic neuroma
labrynthitis
presbyacusis
noise related
drug induced
How well did you know this?
1
Not at all
2
3
4
5
Perfectly
160
Q

conductive hearing loss causes

A
ear wax
acute otitis media
otitis media with effusion
foreign body
cholesteatoma
otosclerosis
How well did you know this?
1
Not at all
2
3
4
5
Perfectly
161
Q

Causes of vertigo

A
BPPV
Labrynthitis
Menieres
Vestibular neuritis 
benign vestibulopathy
How well did you know this?
1
Not at all
2
3
4
5
Perfectly
162
Q

External ear

A

Otitis externa

Malignant otitis externa

How well did you know this?
1
Not at all
2
3
4
5
Perfectly
163
Q

Rinnes and webers

A

If lateralises to the right

  • right conductive
  • left sensorineural

Conductive BC>AC
Sensorineural AC>BC
Normal AC>BC

How well did you know this?
1
Not at all
2
3
4
5
Perfectly
164
Q

Acoustic neuroma presentation

A
Unilateral hearing loss 
Unilateral Tinnitus 
Symptoms of increased ICP
Vertigo
May present with facial nerve palsy
How well did you know this?
1
Not at all
2
3
4
5
Perfectly
165
Q

Ix of acoustic neuroma

A

webers and rinnes
Audiograms
MRI head

How well did you know this?
1
Not at all
2
3
4
5
Perfectly
166
Q

Labrynthitis presentation

A

Sudden onset unilateral hearing loss, tinnitus and vertigo
Nystagmus, vomiting
Recent hx of URTI

How well did you know this?
1
Not at all
2
3
4
5
Perfectly
167
Q

Mx of labrynthitis

A

vestibular suppressants eg prochlorperazine (also anti emetic)

How well did you know this?
1
Not at all
2
3
4
5
Perfectly
168
Q

BPPV presentation

A

Episodes of vertigo after turning head - lasts seconds

Nystagmus, nausea

How well did you know this?
1
Not at all
2
3
4
5
Perfectly
169
Q

BPPV ix and mx

A

Dix-Hallpike then Epley manoeuvre

How well did you know this?
1
Not at all
2
3
4
5
Perfectly
170
Q

Suggested cause of menieres

A

Too much endolymph

How well did you know this?
1
Not at all
2
3
4
5
Perfectly
171
Q

Presentation of menieres

A

Episodes of vertigo, tinnitus, hearing loss lasting mins to hrs
Aural fullness, muffled sound
Nustagmus, nausea

How well did you know this?
1
Not at all
2
3
4
5
Perfectly
172
Q

Mx of menieres

A
Vestibular supressant - prochlorperazine
Betahistidine 
Low salt diet 
Thiazides 
Hearing aids
Surgery
How well did you know this?
1
Not at all
2
3
4
5
Perfectly
173
Q

Presbyacusis presentation

A

bilateral hearing loss
Lose high frequency first - eg female voice, might have TV really loud
Old

How well did you know this?
1
Not at all
2
3
4
5
Perfectly
174
Q

Mx of presbyacusis

A

Social - eg flashing lights instead of doorbell, telephone amplifiers
Hearing aids or cochlear implants

How well did you know this?
1
Not at all
2
3
4
5
Perfectly
175
Q

Ix for hearing loss problems

A

Otoscope
Rinnes and Webers
Audiogram
MRI head

How well did you know this?
1
Not at all
2
3
4
5
Perfectly
176
Q

Pathophys of noise related hearing loss

A

Damage to stereocilia in the cochlear

How well did you know this?
1
Not at all
2
3
4
5
Perfectly
177
Q

Ototoxic drugs

A

Gentamicin

Furosemide

How well did you know this?
1
Not at all
2
3
4
5
Perfectly
178
Q

How to identify which ear on otoscopy image

A

Cone of light is on the same side as the ear

The malleus points away from direction of ear

How well did you know this?
1
Not at all
2
3
4
5
Perfectly
179
Q

What causes otitis media with effusion

A

Negative pressure in the ET tube

How well did you know this?
1
Not at all
2
3
4
5
Perfectly
180
Q

Presentation of otitis media with effusion (glue ear)

A

Developmental delays - speech and learning

Not very responsive to sound

How well did you know this?
1
Not at all
2
3
4
5
Perfectly
181
Q

Otoscopy of glue ear

A

Yellow retracted TM, air bubbles

How well did you know this?
1
Not at all
2
3
4
5
Perfectly
182
Q

Mx of glue ear

A

leave for 3 months as most self resolving

If >3 months - grommets

How well did you know this?
1
Not at all
2
3
4
5
Perfectly
183
Q

Acute otits media pathogens

A

Bacteria - moraxella, haemophillus, pneumococcus

Viral

How well did you know this?
1
Not at all
2
3
4
5
Perfectly
184
Q

Mx of acute otitis media

A

Most likely viral so wait at least 48hrs before giving abx
Fluid, rest, analgesia
Safety Net

How well did you know this?
1
Not at all
2
3
4
5
Perfectly
185
Q

Complications of acute otitis media

A

Mastoiditis then meningitis

Cerebral abcess

How well did you know this?
1
Not at all
2
3
4
5
Perfectly
186
Q

Cholesteatoma pathophys

A

trapped squamous epithelium erodes into the bones

How well did you know this?
1
Not at all
2
3
4
5
Perfectly
187
Q

Causesnof cholesteatoma

A

Recurrent infections paired with ET dysfunction

How well did you know this?
1
Not at all
2
3
4
5
Perfectly
188
Q

Mx of cholesteatoma

A

Semi urgent referral

or urgent if signs of nerve palsy

How well did you know this?
1
Not at all
2
3
4
5
Perfectly
189
Q

Presentation of cholesteatoma

A

Otalgia

Smelly otorrhoea

How well did you know this?
1
Not at all
2
3
4
5
Perfectly
190
Q

Rf for cholesteatoma

A

recurrent infections
down syndrome
Male
low socio economic status

How well did you know this?
1
Not at all
2
3
4
5
Perfectly
191
Q

Complications of cholesteatoma

A

Facial nerve palsy
meningitis
cerebral abscess

How well did you know this?
1
Not at all
2
3
4
5
Perfectly
192
Q

What is otosclerosis

A

fusion of the ossicles, most commonly stapes to oval window

How well did you know this?
1
Not at all
2
3
4
5
Perfectly
193
Q

Common organism of otitis externa

A

pseudamonas

How well did you know this?
1
Not at all
2
3
4
5
Perfectly
194
Q

RF for otitis externa

A

Moist environments eg swimmers
immunocompromised
hearing aids

How well did you know this?
1
Not at all
2
3
4
5
Perfectly
195
Q

Presentation of otitis externa

A

Otalgia - tragus and in mastoid area behind ear

otorrhoea

How well did you know this?
1
Not at all
2
3
4
5
Perfectly
196
Q

complicatons of otitis externa

A

osteomyelitis

temporal bone destruction

How well did you know this?
1
Not at all
2
3
4
5
Perfectly
197
Q

Malignant otitis externa and who is it more common in

A

Otitis externa with its complications + facial nerve palsy

Common in diabetics and CF patients

How well did you know this?
1
Not at all
2
3
4
5
Perfectly
198
Q

Mx of acute sinusitis

A

Send to hospital if has complications
Otherwise wait 10 days, self care measures
If no improvement - corticosteroid nasal spray, back up abx,

How well did you know this?
1
Not at all
2
3
4
5
Perfectly
199
Q

Complications of acute sinusitis

A
Cavernous sinus thrombosis
Cerebral abcesses
Meningitis 
Periorbital cellulitis
Osteomyelitis
How well did you know this?
1
Not at all
2
3
4
5
Perfectly
200
Q

when is sinusitis classed as chronic

A

> 12 weeks

How well did you know this?
1
Not at all
2
3
4
5
Perfectly
201
Q

Causes of chronic sinusitis

A

Kartageners
CF
Facial deformity
Septal deviation

How well did you know this?
1
Not at all
2
3
4
5
Perfectly
202
Q

Mx of chronic sinusitis

A

Nasal douching
Corticosteroid spray
Antihistamines
Macrolides

How well did you know this?
1
Not at all
2
3
4
5
Perfectly
203
Q

Arteries we need to know in the nose

A

Sphenopalatine
Posterior and anterior ethmoid
Kesselbachs area and littles area

How well did you know this?
1
Not at all
2
3
4
5
Perfectly
204
Q

Causes of epistaxis

A
Trauma
HTN
idiopathic
HHT/angiodysplasia
Coagulation disorders
Malignancy 
Unilateral polyp
How well did you know this?
1
Not at all
2
3
4
5
Perfectly
205
Q

Mx of epistaxis

A
A+E if bleeds for >30mins
Anterior packing
Silver nitrate Cautery
Posterior packing
Electrocautery in Surgery
How well did you know this?
1
Not at all
2
3
4
5
Perfectly
206
Q

Red flag nose

A

unilateral bleeding

How well did you know this?
1
Not at all
2
3
4
5
Perfectly
207
Q

Red flag ear

A

unilateral tinnitus

unilateral hearing loss

How well did you know this?
1
Not at all
2
3
4
5
Perfectly
208
Q

Mx of allergic rhinitis

A

antihistamines
steroid nasal spray or oral pred if severe
Nasal irrigation
septoplasty

How well did you know this?
1
Not at all
2
3
4
5
Perfectly
209
Q

Septal haematoma mechanism

A

blunt force trauma
blood between periosteum and perichondrium
reduce blood supply to cartilage
Can end up with saddle nose (or cauliflower ear if pinna haematoma)

How well did you know this?
1
Not at all
2
3
4
5
Perfectly
210
Q

Mx of septal haematoma

A

drain blood

tamponade

How well did you know this?
1
Not at all
2
3
4
5
Perfectly
211
Q

pathogens causing tonsillitis

A

Group A strep
EBV
Viruses eg influenza

How well did you know this?
1
Not at all
2
3
4
5
Perfectly
212
Q

Score to determine if tonsillitis caused by Group A Strep

A

FeverPAIN score

How well did you know this?
1
Not at all
2
3
4
5
Perfectly
213
Q

Abx for bacterial tonsilitis

A

phenoxymethylpenicillin

How well did you know this?
1
Not at all
2
3
4
5
Perfectly
214
Q

Epiglottits presentation

A
4Ds
Distress - respiratory
Drooling
Dysphasia
Dysphonia 

Soft stridor compared to croup which is harsh

How well did you know this?
1
Not at all
2
3
4
5
Perfectly
215
Q

Mx of epiglottitis

A

try and calm them
sedation
intubation
ceftriaxone - as most common cause is haemophillus

How well did you know this?
1
Not at all
2
3
4
5
Perfectly
216
Q

complications of tonsillitis

A

peritonsillar abscess - quinsy

How well did you know this?
1
Not at all
2
3
4
5
Perfectly
217
Q

Bells Palsy mx

A

steroids
eye drops
antivirals eg acyclovir

How well did you know this?
1
Not at all
2
3
4
5
Perfectly
218
Q

Becks triad - what is it and what is it for

A
Cardiac tamponade
Acutely unwell person with
- raised JVP
- hypotension
- muffled heart sounds
How well did you know this?
1
Not at all
2
3
4
5
Perfectly
219
Q

What ECG anomaly can thiazide diuretics such as indapamide cause as a result of their side effect profile

A

flattened t waves as a result of hypokalaemia and hypocalcaemia

How well did you know this?
1
Not at all
2
3
4
5
Perfectly
220
Q

common adverse effects of thiazides

A
dehydration
postural hypotension
hypokalaemia, hypocalcaemia, hyponatraemia
gout
impaired glucose tolerance
impotence
How well did you know this?
1
Not at all
2
3
4
5
Perfectly
221
Q

criteria for Hyperosmolar Hyperglycaemic State (HHS)

A

hypovolaemia
hyperglycaemia >30
serum osmolarity >320

How well did you know this?
1
Not at all
2
3
4
5
Perfectly
222
Q

calculating serum osmolality

A

2xNa + glucose + urea

How well did you know this?
1
Not at all
2
3
4
5
Perfectly
223
Q

Causes of dactylitis

A

spondyloarthropathies eg psoriatic and reactive arthritis, systemic sclerosis
Sickle cell disease
Rare - TB, sarcoidosis, syphillis

How well did you know this?
1
Not at all
2
3
4
5
Perfectly
224
Q

how long are provoked PEs treated for

A

3 months

How well did you know this?
1
Not at all
2
3
4
5
Perfectly
225
Q

what is the first line abx for c diff

A

oral vancomycin

How well did you know this?
1
Not at all
2
3
4
5
Perfectly
226
Q

What is the main antibody test for coeliacs disease

A

anti-TTG

How well did you know this?
1
Not at all
2
3
4
5
Perfectly
227
Q

when do you use rhythm control as first line in AF

A

if reversible cause, coexistent heart failure or if first onset AF

How well did you know this?
1
Not at all
2
3
4
5
Perfectly
228
Q

What tests should adults with suspected asthma have

A

Both

  • fraction of exhaled nitric oxide (FeNO) test
  • spirometry with reversibility testing
How well did you know this?
1
Not at all
2
3
4
5
Perfectly
229
Q

what abx class can cause torsades de pointes

A

macrolides

How well did you know this?
1
Not at all
2
3
4
5
Perfectly
230
Q

sign of LVH on ecg

A

large R waves in V5 and V6 (left sided leads) and deep S waves in V1 and V2 (right sided leads)

How well did you know this?
1
Not at all
2
3
4
5
Perfectly
231
Q

What is a Bartons fracture

A

Intra articular fracture of the distal radius with associated dislocation of the radio-carpal joint

How well did you know this?
1
Not at all
2
3
4
5
Perfectly
232
Q

Presentation of scaphoid fracture

A

Sudden onset pain
pain in anatomical snuffbox
Pain on scaphoid tubercle

How well did you know this?
1
Not at all
2
3
4
5
Perfectly
233
Q

Invs of scaphoid fracture

A

Xray
If none seen repeat at 10-14 days
And then MRI if still no evidence

How well did you know this?
1
Not at all
2
3
4
5
Perfectly
234
Q

mx of undisplaced scaphoid

A

thumb plaster and spica splint
BUT
if fracture of proximal pole then surgery as high risk of avascular necrosis

How well did you know this?
1
Not at all
2
3
4
5
Perfectly
235
Q

mx of displaced scaphoid #

A

percutaneous variable pitch screw for compression

How well did you know this?
1
Not at all
2
3
4
5
Perfectly
236
Q

complications of scaphoid #

A

avn

non-union

How well did you know this?
1
Not at all
2
3
4
5
Perfectly
237
Q

Presentation of carpal tunnel

A

pain and parasthesia of lateral 3 and a half digits
Worse at night
Late stages - may get atrophy of thenar eminence

How well did you know this?
1
Not at all
2
3
4
5
Perfectly
238
Q

Where is spared in carpal tunnel and why

A

Palm

Palmar branch exits before enters beneath flexor retinaculum

How well did you know this?
1
Not at all
2
3
4
5
Perfectly
239
Q

Ddx of carpal tunnel syndrome

A
cervical radiculopathy (C6)
pronator teres syndrome
How well did you know this?
1
Not at all
2
3
4
5
Perfectly
240
Q

Mx of carpal tunnel

A

NSAIDs
Splint (at night)
physio
steroid injections

surgical
- decompression by cutting flexor retinaculum

How well did you know this?
1
Not at all
2
3
4
5
Perfectly
241
Q

what is dupuytrens

A

contraction of longitudinal fascia

How well did you know this?
1
Not at all
2
3
4
5
Perfectly
242
Q

RF of dupuytrens

A
Male
Alcoholic liver cirrhosis 
T1DM
Occupational exposure
Age 40-60
How well did you know this?
1
Not at all
2
3
4
5
Perfectly
243
Q

Pathophys of dupuytrens

A

fibroplastic hyperplasia and altered collagen matrix of palmar fascia

How well did you know this?
1
Not at all
2
3
4
5
Perfectly
244
Q

Typical progression of dupuytrens

A
  • pitting and thickening
  • painless nodules
  • cord contracts
  • flexion deformity
How well did you know this?
1
Not at all
2
3
4
5
Perfectly
245
Q

Test for dupuytrens

A

Heustons test

Lie palm flat - positive if unable to

How well did you know this?
1
Not at all
2
3
4
5
Perfectly
246
Q

Mx of dupuytrens

A

Therapy
Inject Collagenase clostridium histolyticum (NO STEROIDS)

Surgery
- fasciectomy

How well did you know this?
1
Not at all
2
3
4
5
Perfectly
247
Q

What is de quervajns tenosynovitis

A

inflammation of the tendons of the first extensor compartment of the wrist. (by the thumb)

How well did you know this?
1
Not at all
2
3
4
5
Perfectly
248
Q

Presentation of de quervains

A

pain at base of thumb
grasping movements difficult and painful
Finkelstein’s test positive

How well did you know this?
1
Not at all
2
3
4
5
Perfectly
249
Q

Mx of de quervains

A

splint
physio
Steroid injections

Surgical
- decompression of extensor compatment

How well did you know this?
1
Not at all
2
3
4
5
Perfectly
250
Q

What are ganglionic cysts a d where are they found

A

Synovial fluid filled soft tissue lumps caused by degeneration of joints or tendon sheaths
Found along joints and tendons

How well did you know this?
1
Not at all
2
3
4
5
Perfectly
251
Q

Presentation of ganglionic cyst

A

smooth, soft, fluctuant lump along tendon or joint, transilluminates
May have neuro features if compressing nerve

How well did you know this?
1
Not at all
2
3
4
5
Perfectly
252
Q

Mx of ganglionic cysts

A

Usually sort then selves out

If not and rom severely affected, aspirate or excise

How well did you know this?
1
Not at all
2
3
4
5
Perfectly
253
Q

what is trigger finger

A

finger and/or thumb lock or click when in flexion

How well did you know this?
1
Not at all
2
3
4
5
Perfectly
254
Q

pathophys of trigger finger

A

flexor tenosynovitis - due to repetitive movements = inf;lammation

tenosynovitis at metacarpal head (superficial and deep flexor tendons) = nodal formation, distal to the pulley (A1 pulley ligament most common)

moves proximal to pulley when flexed, but cant move back under it on extension = locked in flexed position

How well did you know this?
1
Not at all
2
3
4
5
Perfectly
255
Q

rf of trigger dinger

A

female

age DM

How well did you know this?
1
Not at all
2
3
4
5
Perfectly
256
Q

presentation of trigger finger

A

painful snapping/clicking on finger extension

ca be bilateral and more than one finger

How well did you know this?
1
Not at all
2
3
4
5
Perfectly
257
Q

Mx of trigger finger

A

splint
activity modification
steroid injections

surgical
-percutaneous trigger finger release under local

How well did you know this?
1
Not at all
2
3
4
5
Perfectly
258
Q

What abx can prolong QT interval?

A

Macrolides

How well did you know this?
1
Not at all
2
3
4
5
Perfectly
259
Q

What bloods need to be routinely monitored in the first 12 months of treatment with Simvastatin

A

LFTs

How well did you know this?
1
Not at all
2
3
4
5
Perfectly
260
Q

How long before an op should warfarin be stopped? What is it substituted with? What should INR be less than?

A

at least 5 days before, LMWH

INR should be less than 1.4

How well did you know this?
1
Not at all
2
3
4
5
Perfectly
261
Q

NSAIDs mechanism of action

A

COX inhibitors

Inhibits prostaglandin synthesis from arachidonic acid

How well did you know this?
1
Not at all
2
3
4
5
Perfectly
262
Q

Aspirin MOA

A

inhibits production of thromboxane A by inhibiting both COX1 and 2
Thromboxane A usually enables platelet aggregation

How well did you know this?
1
Not at all
2
3
4
5
Perfectly
263
Q

What does a transaminase (elevated ALT and AST) in the 10 000s commonly indicate

A

paracetamol overdose

How well did you know this?
1
Not at all
2
3
4
5
Perfectly
264
Q

Side effects of calcium channel blockers

A

ankle swelling
headache
flushing

How well did you know this?
1
Not at all
2
3
4
5
Perfectly
265
Q

moa of heparins (both LMWH and unfractionated)

A

Both activate antithrombin 3
LMWH
- increase action of antithrombin 3 on factor Xa so greater inhibition

Unfractionated
- increase action o pf antithrombin 3 on factors Xa, IXa, XIa and XIIa so greater inhibition

How well did you know this?
1
Not at all
2
3
4
5
Perfectly
266
Q

When is unfractionated heparin often used

A

used in situations where there is high risk of bleeding as it can be terminated rapidly. Also useful in renal failure

How well did you know this?
1
Not at all
2
3
4
5
Perfectly
267
Q

What is adhesive capsulitis

A

glenohumeral capsule becomes contracted and adherent to the humeral head

How well did you know this?
1
Not at all
2
3
4
5
Perfectly
268
Q

Rf of adhesive capsulitis

A

Woman
40-60
previous contralateral adhesive capsulitis
DM

How well did you know this?
1
Not at all
2
3
4
5
Perfectly
269
Q

Presentation of adhesive capsulitis

A

general shoulder pain that may radiate to biceps
reduced ROM
Stiffness
Tender to touch

How well did you know this?
1
Not at all
2
3
4
5
Perfectly
270
Q

Mx of adhesive capsulitis

A

Medical

  • physio
  • analgesia
  • Corticosteroid injections

Surgical - after months to years of medical mx

  • manoeuvre joint under GA
  • surgical release of capsule
How well did you know this?
1
Not at all
2
3
4
5
Perfectly
271
Q

Complications of peri operative hypothermia

A

Coagulation - impaired clotting
Infection
Reduced wound healing bc of vasoconstriction
Prolonged recovery from anaesthesia

How well did you know this?
1
Not at all
2
3
4
5
Perfectly
272
Q

Components of the different fluids

A

Hartmanns

  • 131 mmol/L Na
  • 111 Cl
  • 29 HCO3
  • 5 K
  • 2 Ca
  1. 9%NaCl
    - 154 Na
    - 154 Cl

Dextrose
- 278 glucose

How well did you know this?
1
Not at all
2
3
4
5
Perfectly
273
Q

Tonicity and compartment movement of the fluids

A

Hypotonic - hartmanns and NaCl so stay extracellular in vasular and interstitial space
Isotonic - dextrose

How well did you know this?
1
Not at all
2
3
4
5
Perfectly
274
Q

Colloid products

A

FFP
Platelets
Red blood cells

How well did you know this?
1
Not at all
2
3
4
5
Perfectly
275
Q

Why would you give FFP and platelets in a DIC pt

A

Losing them due to clotting

How well did you know this?
1
Not at all
2
3
4
5
Perfectly
276
Q

Daily requirements

A

1mmol/kg/day Na, Cl, K
50-100g per day glucose
25-30ml/kg/day water

How well did you know this?
1
Not at all
2
3
4
5
Perfectly
277
Q

Daily urine output

A

0.5ml/kg/day

How well did you know this?
1
Not at all
2
3
4
5
Perfectly
278
Q

What are the categories of a comprehensive geriatric assessment

A
Medical - problem list, nutrition, meds, co morbidities 
Functional - baseline
Social 
Psychiatric
Environmental
How well did you know this?
1
Not at all
2
3
4
5
Perfectly
279
Q

ADR of dextrose

A

hypokalaemia

How well did you know this?
1
Not at all
2
3
4
5
Perfectly
280
Q

in who would you consider a lower does of opioids for pain mx

A

renal failure as vulnerable to overdose

patients with delirium

How well did you know this?
1
Not at all
2
3
4
5
Perfectly
281
Q

Drugs to be stopped before surgery

A

COCP 4 weeks
Warfarin 5 days
Clopidogrel 7 days

How well did you know this?
1
Not at all
2
3
4
5
Perfectly
282
Q

DVT complications

A
PE
Chronic venous insufficiency 
- lipodermatosclerosis
- venous eczema
- venous ulcer
- hyperpigmentation
How well did you know this?
1
Not at all
2
3
4
5
Perfectly
283
Q

PE complications

A

Death
Type 1 resp failure/hypoxia
RHF
Arrythmias

How well did you know this?
1
Not at all
2
3
4
5
Perfectly
284
Q

Steps taken to reduce the stress of an op on body

A
Oxygenate
Avoid hypothermia
Maintain BP
Nutrition
Analgesia
How well did you know this?
1
Not at all
2
3
4
5
Perfectly
285
Q

What are people on steroids at risk of post op

A

Addisonian crisis

How well did you know this?
1
Not at all
2
3
4
5
Perfectly
286
Q

Why are pts on steroids at increased risk of addisonian crisis

A

HPA axis supression
Cant increase steroids according to demand after trauma
So adrenal insufficiency

How well did you know this?
1
Not at all
2
3
4
5
Perfectly
287
Q

Symptoms and signs of addisonian crisis

A
hypotension
Fatigued
n+v
abdo pain
myalgia
hyponatraemia
hypoglycaemia
How well did you know this?
1
Not at all
2
3
4
5
Perfectly
288
Q

mx of addisonian crisis

A

immediate resus with IV hydrocortisone

How well did you know this?
1
Not at all
2
3
4
5
Perfectly
289
Q

Mx of diabetes perioperatively

A

T1DM

  • stop sc insulin
  • use variable rate
  • NBM
  • regular monitoring of BM
  • first on theatre list
  • maintenance fluid - generally 5% dextrose
How well did you know this?
1
Not at all
2
3
4
5
Perfectly
290
Q

what effect does stress have on goucose levels

A

hyperglycaemic

How well did you know this?
1
Not at all
2
3
4
5
Perfectly
291
Q

Components of MUST score

A

BMI
Unintentional weight loss percentage over last 3-6months
Acutely unwell and no nutritional intake for >5 days

How well did you know this?
1
Not at all
2
3
4
5
Perfectly
292
Q

What blood type is universal donor and what is universal recipient

A

Donor - O

Recipient - AB

How well did you know this?
1
Not at all
2
3
4
5
Perfectly
293
Q

Difference between G+S and crossmatch

A

G+S identifies blood group - there is no issuing of blood.

Cross match identifies if any immune reactions with matched units of blood and then issues x amount of blood bags

How well did you know this?
1
Not at all
2
3
4
5
Perfectly
294
Q

When would you halve the dose of prophylactic dalteparin

A

if eGFR <30

How well did you know this?
1
Not at all
2
3
4
5
Perfectly
295
Q

referral criteria for possible colorectal cancer

A
>40 with unexplained weight loss and abdo pain
>50 with unexplained rectal bleeding
>60 with any of 
- iron deficiency anaemia
- changes in bowel habit
<50 with rectal bleeding AND any of the following unexplained symptoms
- abdo pain
- change in bowel habit
- weight loss
- iron deficiency anaemia
How well did you know this?
1
Not at all
2
3
4
5
Perfectly
296
Q

Dukes classification for colorectal cancer

A

A limited to bowel
B extending through bowel wall (beyond the muscularis)
C regional lymph node involvement
D distant metastases

How well did you know this?
1
Not at all
2
3
4
5
Perfectly
297
Q

Complications of stomas

A

Early

  • bowel ischaemia/necrosis
  • bowel retraction
  • parasternal abscess
  • poor stoma functtion
  • high output

Late

  • parastomal hernia
  • stenosis
  • prolapse
  • adhesion leading to BO
  • parastomal dermatitis
  • bowel dysmobility
  • malnutrition
  • psychosocial complications
How well did you know this?
1
Not at all
2
3
4
5
Perfectly
298
Q

Chronic mesenteric ischaemia presentation

A

Older people with rf of arterial disease
Diffuse colicky abdo pain worse after eating
Weight loss due to pain as aboid eating
May have diarrhoea and malaena or haematochezia
Abdo tenderness
Epigastric bruits may be present

How well did you know this?
1
Not at all
2
3
4
5
Perfectly
299
Q

Indications for liver transplant after paracetamol imduced liver failure

A

pH <7.3 24 hrs after ingestion
prothrombin time >100s
creatinine >300
grade 3 or 4 encephalopathy

How well did you know this?
1
Not at all
2
3
4
5
Perfectly
300
Q

Mx of paracetamol overdose

A

less than 1 hr - activated charcoal
Otherwise acetylcysteine
(wait 4 hrs after ingestion to get paracetamol level)

How well did you know this?
1
Not at all
2
3
4
5
Perfectly
301
Q

extra intestinal features of IBD

A

Dermatology - erythema nodosum, pyoderma gangrenosum
Ocular - anterior uveitis, episcleritis, conjunctivitis
MSK - assymetrical arthritis
HPB - primary sclerosing cholangitis (more common in UC)
AA amyloidosis

How well did you know this?
1
Not at all
2
3
4
5
Perfectly
302
Q

Colonoscopy and biopsy results in UC

A

continuous inflammation with an erythematous mucose, loss of haustra and pseudopolyps

Biopsy - loss of goblet cells, crypt abscess and inflammatory cells (predominantly lymphocytes)

How well did you know this?
1
Not at all
2
3
4
5
Perfectly
303
Q

mx of acute UC

A

1 - IV corticosteroids
2 - Add IV cyclosporin or consider surgery
3 surgery (indications are toxic megacolon, acute fulminant UC)

How well did you know this?
1
Not at all
2
3
4
5
Perfectly
304
Q
1 Ivor Lewis oesophagectomy
2 Nissen fundoplication
3 gastrectomy
4 lap chole
5 whipples pancreaticoduodenectomy
6 Hartmanns
A
1- oesophageal cancer
2 - GORD/hiatus hernia
3 - gastric cancer, Zollinger ellison
4 - gall stones
5 - pancreatic cancer
6 - diverticulitis, LBO, (resection of rectosigmoid colon)
How well did you know this?
1
Not at all
2
3
4
5
Perfectly
305
Q

inducing remission in UC pts first line drugs

A

5ASA’s – sulfasalazine

2nd line prednisolone

How well did you know this?
1
Not at all
2
3
4
5
Perfectly
306
Q

Hyperkalaemia ve early STEMI ECG changes

A

Both have hyperacute t waves

Hyperkalaemia would be widespread, stemi would be regional

How well did you know this?
1
Not at all
2
3
4
5
Perfectly
307
Q

Presentation of a heart attack in a female with poor diabetic control

A

Atypical
Mild non specifc upper abdo pain
Central autonomic neuropathy so may not get sweating and tachycardia

How well did you know this?
1
Not at all
2
3
4
5
Perfectly
308
Q

complications of RCA occlusion

A

Supplies SAN and AVN so can get bradycardia/heart block

How well did you know this?
1
Not at all
2
3
4
5
Perfectly
309
Q

Leads of the ecg and territories

A

Inferior - 2,3 and avF
Lateral - 1,avL and V5 V6
Anterior - V1-V4
Septal - V1-V2

How well did you know this?
1
Not at all
2
3
4
5
Perfectly
310
Q

what can you hear on auscultation of heart in a stemi

A

4th heart sound

How well did you know this?
1
Not at all
2
3
4
5
Perfectly
311
Q

ECHO finding on STEMI

A

regional wall muscle abnormality - not contracting properly

How well did you know this?
1
Not at all
2
3
4
5
Perfectly
312
Q

pulmonary oedema signs on CXR

A

kerley b lines - horizontal lines at bases
bat wing sign
may have increased cardio thoracic ratio if cardiogenic in cause

How well did you know this?
1
Not at all
2
3
4
5
Perfectly
313
Q

mx of STEMI

A
ROMANCE
Reassurance
Oxygen
Morphine and Metoclopramide 
Aspirin 300mg PO
Nitrates - GTN
Clopidogrel/prasugrel 
Enoxaparin or another LMWH 

Then PCI

Then Meds for life
Aspiring 75mg
ACEi
Beta blocker 
Clopidogrel - for first 12 months 
Statin
How well did you know this?
1
Not at all
2
3
4
5
Perfectly
314
Q

Mx of NSTEMI

A
How well did you know this?
1
Not at all
2
3
4
5
Perfectly
315
Q

GTN action

A

vasodilates (risk of hypotension)

Venodilates - reduces pre load of heart and increases blood flow to heart

How well did you know this?
1
Not at all
2
3
4
5
Perfectly
316
Q

Complications of MI

A
DREAD
Death
Rupture eg of papillary muscles
oEdema
Arrythmias
Dresslers - post MI syndrome - get pericarditis about 2 weeks after MI
How well did you know this?
1
Not at all
2
3
4
5
Perfectly
317
Q

ECG of pericarditis

A

global ST elevation - saddle back

PR depression

How well did you know this?
1
Not at all
2
3
4
5
Perfectly
318
Q

Signs of cardiac tamponade

A

Becks triad

  • hypotension
  • Increased JVP
  • muffled heart sounds
How well did you know this?
1
Not at all
2
3
4
5
Perfectly
319
Q

Tx of cardiac tamponade

A

pericardiocentesis

How well did you know this?
1
Not at all
2
3
4
5
Perfectly
320
Q

mx of SVT

A

vagal manoeuvre
Adenosine (up to 3 times)
synchronised DC cardioversion if haemodynamically unstabel and above not worked

How well did you know this?
1
Not at all
2
3
4
5
Perfectly
321
Q

causes of VT

A

MI

Structural heart disease

How well did you know this?
1
Not at all
2
3
4
5
Perfectly
322
Q

cause of torsades de pointes

A

long QT - caused by drugs

How well did you know this?
1
Not at all
2
3
4
5
Perfectly
323
Q

digoxin toxicity ecg findings

A

ventricular ectopics,
AF
Bradycardias
Reverse tick sign

How well did you know this?
1
Not at all
2
3
4
5
Perfectly
324
Q

if arrythmia cant be controlled what is the mx

A

ICD or pacemaker

How well did you know this?
1
Not at all
2
3
4
5
Perfectly
325
Q

mobitz type 1 vs 2

A

type 1 increases

type 2 stays the same - higher risk of going into heart block

How well did you know this?
1
Not at all
2
3
4
5
Perfectly
326
Q

Where best to hear murmurs

A

aortic stenosis - aortic area and carotids
aortic regurge - er s when leaning forward and inspiring
mitral regurge - axilla
Mitral stenosis - apex

How well did you know this?
1
Not at all
2
3
4
5
Perfectly
327
Q

Ix of hf

A

ECHO
ECG
pro-BNP
CXR

How well did you know this?
1
Not at all
2
3
4
5
Perfectly
328
Q

pink frothy sputum supign of what…

A

pulmonary oedema

How well did you know this?
1
Not at all
2
3
4
5
Perfectly
329
Q

normal ejection fracture

A

greater than 50%

How well did you know this?
1
Not at all
2
3
4
5
Perfectly
330
Q

cause of raised bnp in copd

A

cor pulmonale

How well did you know this?
1
Not at all
2
3
4
5
Perfectly
331
Q

Score used to classify heart failure

A

New York

How well did you know this?
1
Not at all
2
3
4
5
Perfectly
332
Q

mx of hf

A

Symptom control

  • loop diuretics eg furosemide
  • GTN

Reduced mortality

  • beta blockers
  • ACEi/ARBSx
How well did you know this?
1
Not at all
2
3
4
5
Perfectly
333
Q

Which valvular problem can cause AF

A

Mitral stenosis

How well did you know this?
1
Not at all
2
3
4
5
Perfectly
334
Q

side effect of hartmanns solution and when not to use

A

hyperkalaemia and lactic acidosis

with care in sepsis mx

How well did you know this?
1
Not at all
2
3
4
5
Perfectly
335
Q

Rules of fluid resus

A

give 500ml bolus unless HF/very fail (250ml) or pancreatitis (1L)
Not hartmanns if lactate or hyperkalaemic

How well did you know this?
1
Not at all
2
3
4
5
Perfectly
336
Q

Rules of potassium in maintenance fluids

A

max 40mmol per bag
max 10mmol/hr of KCL
max 80ml a day

How well did you know this?
1
Not at all
2
3
4
5
Perfectly
337
Q

target INR pre surgery for pt on warfarin

A

<1.5

How well did you know this?
1
Not at all
2
3
4
5
Perfectly
338
Q

Complications of meningits

A
How well did you know this?
1
Not at all
2
3
4
5
Perfectly
339
Q

Complications of meningits

A
Septic shock
DIC
Seizures
Coma
Subdural effusions 
Death
SIADH

Abscess
Hearing loss

How well did you know this?
1
Not at all
2
3
4
5
Perfectly
340
Q

organisms causing menitngitis

A
Neisseria meningitidis
Streptococcus pneumoniae
Haemophilus influenza
Listeria
Measles and rubella
How well did you know this?
1
Not at all
2
3
4
5
Perfectly
341
Q

Abx for streps

A

Group A, viridans - benpen

Pneumonia - amoxicllin, clarithromycin, IV cephalosporin

How well did you know this?
1
Not at all
2
3
4
5
Perfectly
342
Q

Abx for MRSA

A

glycopeptide such as vancomycin

How well did you know this?
1
Not at all
2
3
4
5
Perfectly
343
Q

abx for sepsis

A

meropenem

or as per local guidelines

How well did you know this?
1
Not at all
2
3
4
5
Perfectly
344
Q

abx for c diff

A

metronidazole or vancomycin

How well did you know this?
1
Not at all
2
3
4
5
Perfectly
345
Q

sepsis definition

A

life threatening organ dysfunction caused by dysregulated host response to infection

How well did you know this?
1
Not at all
2
3
4
5
Perfectly
346
Q

septic shock definition

A

a subset of sepsis with profound circulatory cellular and metabolic abnormalities associated with greater risk of mortality than sepsis alone

How well did you know this?
1
Not at all
2
3
4
5
Perfectly
347
Q

Red flags of sepsis

A
resp rate >25 
HR>130
New need for greater than 40% O2 to keep sats over 91%
systolic BP <90 or fall of 40 from normal
No urine output for 16hrz
New onset delirium
Non blanching rash 
Neutropenia or chemo in last 6 weeks
How well did you know this?
1
Not at all
2
3
4
5
Perfectly
348
Q

Which malaria organism causes most severe form of malaria

A

o falciparum

How well did you know this?
1
Not at all
2
3
4
5
Perfectly
349
Q

presentation of malaria

A

abrupt onset of rigors followed by high fever, malaise, severe headache and myalgia
Vague abdo pain, n+v

How well did you know this?
1
Not at all
2
3
4
5
Perfectly
350
Q

signs of malaria

A

jaundice

hepatosplenomegaly

How well did you know this?
1
Not at all
2
3
4
5
Perfectly
351
Q

presentation of typhoid

A
sustained fever
rose spots 
anorexia
malaise
vague abdo discomfort
constipation or diarrhoea 
dry cough
How well did you know this?
1
Not at all
2
3
4
5
Perfectly
352
Q

signs of typhoid

A

hepatosplenomegaly
rose spots
pulse temperature dissociation

How well did you know this?
1
Not at all
2
3
4
5
Perfectly
353
Q

definition/criteria of PUO

A

temp >38 degrees on multiple occasions
illness >3 weeks
No diagnosis despite 1 week worth of inpatiemt

How well did you know this?
1
Not at all
2
3
4
5
Perfectly
354
Q

screening for latent TB

A

cxr and quantiFERON (measurement of interferon gamma)

How well did you know this?
1
Not at all
2
3
4
5
Perfectly
355
Q

can quantiFERKN differentiate between active and latent TB

A

no

How well did you know this?
1
Not at all
2
3
4
5
Perfectly
356
Q

who is screened for TB

A

healthcare workers
Immigrants from high prevalance countries
HIV positive pts
patients starting on immunosupression

How well did you know this?
1
Not at all
2
3
4
5
Perfectly
357
Q

treatment of latent TB

A

3 months Rifampicin and isoniazid
pts aged >35 are at increased risk of hepatotoxicity so advised against treating TB in these pts unless they have other risk factors eg HIV

How well did you know this?
1
Not at all
2
3
4
5
Perfectly
358
Q

Active TB presentation

A

non resolving cough
unexplained persistent fever
Drenching night sweats
weight loss

How well did you know this?
1
Not at all
2
3
4
5
Perfectly
359
Q

gold standard for TB ix

A

cultures of sputum

How well did you know this?
1
Not at all
2
3
4
5
Perfectly
360
Q

paradoxical reaction of TB treatment

A

increased inflammation as bacteria die causing symptom worsening
If TB is affecting sites where swelling cant be tolerated eg meningeal/spinal/pericardial, then steroids are given at the start of treatment

How well did you know this?
1
Not at all
2
3
4
5
Perfectly
361
Q

what further ix should those with miliary tb have and why

A

CT/MRI head and LP to exclude CNS involvement

How well did you know this?
1
Not at all
2
3
4
5
Perfectly
362
Q

where should TB pts be managed

A

in negative pressure side rooms and staff should use PPE

How well did you know this?
1
Not at all
2
3
4
5
Perfectly
363
Q

Baseline ix for pts newly dx with HIV

A

Confirmatory HIV test
CD4 count
HIV viral load
HIV resistance profile
HLA B*5701 status
Serology for syphilis, hepatitis B (sAg, cAb, sAb), hepatitis C, hepatitis A
Toxoplasma IgG, measles IgG, varicella IgG, rubella IgG
FBC, U&Es, LFTs, bone profile, lipid profile
 Schistosoma serology (if has spent >1 month in sub-Saharan Africa)
 Women should have annual cervical cytology.

How well did you know this?
1
Not at all
2
3
4
5
Perfectly
364
Q

What opportunistic infections are pts with a CD4 count <200 at risk of

A

PCO
MAI
CMV

How well did you know this?
1
Not at all
2
3
4
5
Perfectly
365
Q

what vaccinations should HIV pts recieve

A

Hep B and pneumococcus

Annual influenza vaccine

How well did you know this?
1
Not at all
2
3
4
5
Perfectly
366
Q

Needle stick injury guidelines

A
bleed wound
wash with soap and water
May also use antiseptic 
Contact OH or ED if out of hrs
need for post exposure prophylaxis (PEP) will be assessed
How well did you know this?
1
Not at all
2
3
4
5
Perfectly
367
Q

broad spectrum abx

A

Ceftriaxone
Clarithromycin
meropenem
Amoxicillin

How well did you know this?
1
Not at all
2
3
4
5
Perfectly
368
Q

best ix for gonorrhoea and chlamidyia

A

NAAT - urine nucleic acid amplification test

How well did you know this?
1
Not at all
2
3
4
5
Perfectly
369
Q

top 2 causes of hypercalacaemia

A

primary hyperparathyroidism eg parathyroid adenoma

cancers eg lung cancer releasing PTHrP, or bone mets,

How well did you know this?
1
Not at all
2
3
4
5
Perfectly
370
Q

causes of hypercapcaemia with a low PTH

A

cancers
TB
sarcoidosis

How well did you know this?
1
Not at all
2
3
4
5
Perfectly
371
Q

why get hypercalcaemia in CKD

A

renal bone disease

parathyroid gland hypertrophy as a result of tertiary hyperparathyroidism

(Calcium levels start low but eventually raise as PTH levels go crazy)

How well did you know this?
1
Not at all
2
3
4
5
Perfectly
372
Q

what diuretic causes raised calcium

A

thiazides

How well did you know this?
1
Not at all
2
3
4
5
Perfectly
373
Q

presentation of hypercalcaemia

A
stones
moans
bones 
groans 
dehydration, polydipsia
How well did you know this?
1
Not at all
2
3
4
5
Perfectly
374
Q

effect of hypecalcaemia on ecg

A

shortens QT so can get arrythmias

How well did you know this?
1
Not at all
2
3
4
5
Perfectly
375
Q

hypercalcaemia mx

A

large amounts of fluids to rehydrate

Bisphosphonates

How well did you know this?
1
Not at all
2
3
4
5
Perfectly
376
Q

which diuretics lower calcium

A

loop diuretics

How well did you know this?
1
Not at all
2
3
4
5
Perfectly
377
Q

ix for hypercalcaemia

A

calcium
PTH
Parathyroid ultrasound

search for underlying cause - think cancer, kidneys,

How well did you know this?
1
Not at all
2
3
4
5
Perfectly
378
Q

causes of hypocalcaemia

A
autoimmune destruction of parathyroid 
surgery damage
radiation
tumour
low magnesium
prostate cancer (osteoblastic)
How well did you know this?
1
Not at all
2
3
4
5
Perfectly
379
Q

causes of low calcium and high PTH (secondary hyperparathyroidism)

A

poor diet
malabsorption
reduced vit D
early ckd stages

How well did you know this?
1
Not at all
2
3
4
5
Perfectly
380
Q

presentation of hypocalcaemia

A
Parasthesia in hands and lips
Chvosteks sign 
carpo pedal spasm
tetany
abdo cramping 
prolonged QT
seizures and laryngospasm - complications to prevent
Diarrhoe 
How well did you know this?
1
Not at all
2
3
4
5
Perfectly
381
Q

mx of hypocalcaemia

A

IV or oral calcium depending on severity

treat caues eg give magnesium or vit d

How well did you know this?
1
Not at all
2
3
4
5
Perfectly
382
Q

causes of hyperkalaemia

A

renal - reduce excretion

  • spirinolactone
  • ACEi/ARBs
  • NSAIDs (can worsen AKI)
  • AKI/CKD

increased intake - diet

extracellular shift

  • hypoglycaemia
  • DKA
  • rhabdomyolysis
  • tumour lysis
How well did you know this?
1
Not at all
2
3
4
5
Perfectly
383
Q

cause of pseudonyperkalaemia

A

leaving tourniquet on too long or too tight

How well did you know this?
1
Not at all
2
3
4
5
Perfectly
384
Q

what type of arrythmia does hyperkalaemia cause

A

bradyarrythmias

How well did you know this?
1
Not at all
2
3
4
5
Perfectly
385
Q

what is classed as severe hyperkalaemia

A

> 6.5

or with any ecg changes

How well did you know this?
1
Not at all
2
3
4
5
Perfectly
386
Q

mx of hyperkalaemia

A

protect heart - slow 10% IV calcium gluconate 30ml
Drive into cells - 10 units insulin (actrapid) in 50ml 5% dextrose in 100ml bag of saline, or 5mg Neb salbutamol
Clear from gut - calcium resonium

if doesnt work, ITU and haemodialysis

How well did you know this?
1
Not at all
2
3
4
5
Perfectly
387
Q

hypokalaemia causes

A
loops and thiazides 
vomiting and diarrhoea 
decreased intake 
intracellular shift - alkaline, insulin, salbutamol, 
laxatives
hyperaldosteronism
How well did you know this?
1
Not at all
2
3
4
5
Perfectly
388
Q

gi complication of low potassium

A

paralytic ileus

How well did you know this?
1
Not at all
2
3
4
5
Perfectly
389
Q

what toxicity can hypokalaemia aggravate

A

digoxin

How well did you know this?
1
Not at all
2
3
4
5
Perfectly
390
Q

causes of hypernatraemia

A

hyperaldosteronism
dehydration - d+v, sweating, burns
conns syndrome - raised aldosterone
cushings - cortisol can mimic aldosterone in high concentrations

ix using serum and urine osmolality

How well did you know this?
1
Not at all
2
3
4
5
Perfectly
391
Q

mx of hypernatraemia

A

give fluids to dilute sodium

How well did you know this?
1
Not at all
2
3
4
5
Perfectly
392
Q

what can happen if correct hypernatraemia to quickly

A

cerebral oedema

How well did you know this?
1
Not at all
2
3
4
5
Perfectly
393
Q

hyponatraemia causes

A
SIADH 
SSRIs
primary polydypsia
thiazides
addisons

fluid overload- HF, CKD, liver cirrhosis

How well did you know this?
1
Not at all
2
3
4
5
Perfectly
394
Q

epinephrine

A

IM 0.5mg (1in1000) every 5 mins

How well did you know this?
1
Not at all
2
3
4
5
Perfectly
395
Q

Hydrocortisone

A

IV 100mg

How well did you know this?
1
Not at all
2
3
4
5
Perfectly
396
Q

Chlorphenamine

A

IM or IV 10mg

How well did you know this?
1
Not at all
2
3
4
5
Perfectly
397
Q

Atropine

A

IV 500 micrograms

How well did you know this?
1
Not at all
2
3
4
5
Perfectly
398
Q

Aspirin

A

PO 300mg

How well did you know this?
1
Not at all
2
3
4
5
Perfectly
399
Q

clopidogrel

A

300mg PO prior to PCI

How well did you know this?
1
Not at all
2
3
4
5
Perfectly
400
Q

calcium gluconate

A

IV 30ml 10%

How well did you know this?
1
Not at all
2
3
4
5
Perfectly
401
Q

Insulin dextrose infusion

A

10 units of insulin in 50ml 5% dextrose in 100ml normal saline

How well did you know this?
1
Not at all
2
3
4
5
Perfectly
402
Q

diazepam

A

IV 10mg or PR 10mg

How well did you know this?
1
Not at all
2
3
4
5
Perfectly
403
Q

Lorazepam

A

IV 4 mg

How well did you know this?
1
Not at all
2
3
4
5
Perfectly
404
Q

salbutamol

A

Neb 5mg

How well did you know this?
1
Not at all
2
3
4
5
Perfectly
405
Q

prednisolone in asthma

A

PO 40mg

How well did you know this?
1
Not at all
2
3
4
5
Perfectly
406
Q

morphine

A

PO 10mg or IV 5mg

Given with metoclopramide also 10mg

M+M 10+10

How well did you know this?
1
Not at all
2
3
4
5
Perfectly
407
Q

Naloxone

A

IV 400 micrograms

How well did you know this?
1
Not at all
2
3
4
5
Perfectly
408
Q

describing lesion acronym

A
SCAM
Site/distribution
Colour/configuration
Associated features
Morphology (shape)
409
Q

BCC presentation

A

shiny pearly rolled edge
Talengiectasia
commonly occurs in head and neck

410
Q

rf of bcc

A

previous skin cancer
sun exposure - sun beds or frequently burned
fhx
immunosuppressed

411
Q

referall for BCC

A

routine referral unless near eye or 2 mm from nasolabial folds or immunosupressed then urgent 2 ww

412
Q

skin scc presentation

A
ulcerated
bleeding
crusty
oozing
painful
413
Q

ddx for melanoma

A

sebharroeic keratosis

414
Q

glasgow score for melnoma

A
major
change in 
-size
-shape
-colour

minor

  • diameter >7mm
  • oozing
  • change in sensation
  • looks inflammed
415
Q

gp conservative mx of skin cancers

A

advise against sun bed use

regularly check skin

416
Q

what is actinic keratosis? what is it a risk factor for?

A

Old man bald head skin lesion

can predispose to skin scc

417
Q

cellulitis description

A

confluent erythematous rash

418
Q

mx of impetigo

A

topical fusidic acid

419
Q

what to say for mx

A

safety net

420
Q

varicella zoster (shingles) treatment within 72hrs

A
give acyclovir, doesmt work after
stay away from pregnant lady
avoid people who havent had it 
antihistamines and calamine lotion
wear mittens
421
Q

fetal varicella syndrome

A

can cause developmental delays and deformities

422
Q

Causes of hyperparathyroidism

A

Primary
- solitary pituitary adenoma (80%)

Secondary

  • chronic renal insufficiency
  • vit D deficiency
422
Q

Causes of hyperparathyroidism

A

Primary
- solitary pituitary adenoma (80%)

Secondary

  • chronic renal insufficiency
  • vit D deficiency
423
Q

Mx of hyperparathyroidism

A

Treat cause eg surgical excision of adenoma
Vit D replacement
Phosphate binders

424
Q

Causes of hypoparathyroidism

A

Autoimmune
Congenital - Di George, fallot
Iatrogenic - surgery, radiation

425
Q

Presentation of hypoparathyroidism

A
low calcium
Spasms
Tetany
Carpo pedal spasm 
Chovsteks 
seizures
426
Q

mx of hypoparathyroidism

A

ca supplements

calcitriol

426
Q

mx of hypoparathyroidism

A

ca supplements

calcitriol

427
Q

What is pseudohyperparathyroidism

A

PTH resistance due to mutated G protein

428
Q

phaechromocytoma presentation

A
paroxysmal hypertension
sweating
palpitations 
pale - bc adrenaline vasoconstricts
headaches
429
Q

what hormones do phaechromocytoma secret

A

Adrenaline

Some can secrete EPO so get polycythaemia

430
Q

blood test for phaeo

A

metanephrines

431
Q

mx of phaechromocytoma

A

alpha blocker! Then beta blockers then adrenalectomy

432
Q

side effect of tamsulosin in older people to be aware of

A

hypotension. its an alpha blocker

433
Q

example of mineral clrticoid

A

aldosterone

434
Q

what inhibits prolactin

A

dopamine

435
Q

na and k in addisons

A

low sodium

high potassium

436
Q

addisons ix

A

9am cortisol

SynACHTen - differentiates whether it is primary or secondary

437
Q

what enzyme attacks adrenal gland in autoimmune addisons

A

21 hydroxylase

438
Q

3S’s of treating addisonian crisis

A

Steroids - IV hydrocortisone
Sugar - as hypoglycaemic
Saline - hypotensive

439
Q

what will pt need to be given for long term steroid use

A

steroid emergency card

medical wristband

440
Q

Side effects of long term steroid use

A
Cataracts
Ulcers
Striae
Hypertension, hirsutism
Infection
Necrosis
GI upset
Osteoporosis
Insomnia
Diabetes
441
Q

Advise for long term steroid users

A

increase dose when ill
wear steroid emergency bracelet
dont stop abruptly

442
Q

what female drugs can affect cortisol levels

A

any containing oestrogen. Check if they are on it if have elevated cortisol

443
Q

What is conns

A

adrenal tumour secreting aldosterone

444
Q

causes of cushings

A

exogenous steroid use
Small cell lung cancer
cushings disease - pituitary adenoma
Cortisol releasing adrenal tumour

445
Q

why can you get hyperpigmentation in cushings disease

A

bc releases acth which mimics ACTH

446
Q

mx of cushings

A

depends on cause

mentiripone

447
Q

Acromegaly ix and common cause

A

Pituitary adenoma

OGTT - bc GH increases insulin resistance
IGF1
MRI head

448
Q

sx and signs of acromegaly

A
Clumsy
Big feet
big hands
deep voice
teeth gap
underbite
carpal tunnel at night 
prominent forehead
449
Q

what cancer is associated with acromegaly

A

colorectal

450
Q

commonest cause of goitre worldwide

A

iodine deficiency

451
Q

causes of goitre with normal tfts

A

menarche
pregnancy
menopause

452
Q

antibodies in graves

A

tsh stimulating ab

453
Q

presentation of graves

A
relapsing remitting 
palpitations 
heat intolerance and weight loss
agitation
!anxiety
exopthalmos and lid lag 
pretibal myxedema
thyroid acropachy
454
Q

what heart drug can cause hyperthyroidism

A

amiodarone

455
Q

goitre and pain

A

thyroiditis - de quervains

456
Q

mx of hyperthyroid

A

block and replace - high dose carbomazole + thyroxine

457
Q

bad side effect if carbimazole

A

agranulocytosis - lowers neutrophils so increased risk of infection. may present with tonsilitis
do urgent fbc

458
Q

hashimotos ab

A

anti - TPO

459
Q

shehan syndrome

A

pregnancy and haemorrhage

460
Q

hypothyroidism presentation

A
eyelid swelling
hoarseness
bradycardia
weight gain
cold intolerence
461
Q

svever complication of hypothyoodism

A

mxodema coma

462
Q

calculating anion gap

A

Na - (Cl + HCO3)

463
Q

When will you get metabolic acidosis with a normal anion gap

A

If there is a reduced alkali

  • GI losses such as vomiting
  • renal losses such as renal tubular acidosis, addisons
  • toxins
464
Q

when will you get a metabolic acidosis with high anion gap

A

acidosis due to acid increase

  • Lactic acidosis eg exercise, sepsis
  • ketoacidosis - diabetes
  • toxins
  • renal failure
465
Q

how to differentiate CKD and AKI on U+Es

A

Urea high in AKI, can be normal in CKD (but can get uraemia?)
eGFR drops suddenly in AKI, gradual decline in CKD

466
Q

AKI staging

A

Serum Creatinine. Urine output
1 - 1.5-1.9x baseline. <0.5ml/kg/hr for 6-12hrs
2 - 2 - 2.9x baseline. <0.5ml/kg/hr for 12-24hrs
3 - 3x baseline. <0.3ml/kg for >24hrs OR anuria for 12hrs

467
Q

Rf for AKI

A
dehydration
elderly 
hospital stay
DM
CKD
Sepsis
Nephrotoxic meds
468
Q

Causes of AKI

A

Pre renal - hypovolaemia, renal artery stenosis, reduced CO, meds
Renal - sepsis, vasculitis, acute glomerular nephritis
Post renal - bladder outflow obstruction, B/L pelvicoreteral blockage

469
Q

Indications for RRT in AKI

A
hyperkalaemia persisting after therapy
metabolic acidosis persisting after therapy
uraemic encephalitis
uraemic pericarditis
fluid overload resistant to diuretics
AEIOU
Acidosis
Electrolytes
Intoxicants
Overload
Uraemia
470
Q

causes of ckd

A
hypertension
dm
renovascular disease
chronic/recurrent pyelonephritis
glomerulonephritis
471
Q

complications of CKD

A

renal mineral bone disease
Anaemia of chronic disease
Hyperparathyroidism

472
Q

Pathophys of mineral bone disease in ckd

A

reduced calcidiol to calcitriol (activated Vit D) as happens in kidney so less gut absorption of ca.
Increased absorption from bones
also get hyperparathyroidism

473
Q

pathophys of anaemia in CKD

A

reduced EPO production
Anaemia of chronic disease
uraemia can cause bone marrow suppression
functional B12 deficiency

474
Q

classification of lateral malleolus fractures

A

Weber
A- Below sydesmosis
B- At syndesmosis
C- Above syndesomosis

475
Q

Which weber fracture is most unstable

A

Weber C

Proximal ankle fractures are more unstable than distal

476
Q

Ottawa ankle rules are used when diagnostic uncertainty. They indicate when an xray must be done in the presence of what 3 features?

A

bone tenderness at posterir edge or tip of lateral malleolus
OR
bone tenderness at posterior edge or tip of the medial malleolus
OR
an inability to weight bear both immediately and in the emergency department for 4 steps

477
Q

when cant ottawa ankle rules be used

A

intoxicated pt
uncooperative pt
distracting painful injuries else where
diminished sensation in legs

478
Q

what position should an ankle be xrayed in and why

A

dorsi flexion ( if plantar it can appear to have talar shift)

479
Q

how is leg positioned for a mortise view1

A

5-20 degrees internal rotation

480
Q

what needs to be lax for talar shift

A

deltoid ligament

481
Q

Immediate Mx of ankle fracture

A

immediate reduction
then ankle below knee back slab
post reduction neurovascular exam
repeat plain film radiograph

482
Q

Conservative mx of ankle fractures and in who

A

non displaced medial malleolus fractures or Weber A and B.

Cast for pain relief, weight bearing, re xray 5-7 days after

483
Q

surgical mx of xray fractures and in who

A

Weber C
Weber B with talar shift
displace bimalleolar or trimalleolar fractures
open fractures

ORIG

484
Q

Complications of ankle fractures

A

post traumatic OA
mal union
infection
haemorrhage blah blah blah

485
Q

MOI of calcaneal fracture

A

jump from height, axial loading

486
Q

why is there a high risk of avascular necrosis in a talar fracture

A

most commonly fractures through neck

has extra osseous arterial supply which is susceptible to interruption

487
Q

ddx of ankle fractures caused by jumping from height

A

talar fracture
calcaneal fracture
tibial pilon/plafond fracture
tibial plateau

488
Q

mx options for ankle OA

A

analgesia
activity modification
limit movement in an ankle brace

Surgical - realignement osteotomy, arthrodesis (but lose plantar flexion), replacement (but revision is poor)

489
Q

low ankle sprain is injury to what ligaments

A
calcaneofibular ligament (most common)
anterior talofibular ligaments
490
Q

mx of halux valgus

A

change shoes
spacer socks
braces
surgery - metatarsal osteotomy

491
Q

blood supply to NOF

A

medial circumflex artery - lies on the intracapsular femoral neck

492
Q

classic appearance of NOF#

A

shortened externally rotated

493
Q

ddx of NOF#

A

pelvic (pubic ramus) fractures
acetabular fracture
femoral diaphysis fracture
femoral head fracture

494
Q

what imaging would you do if suspected a pathological NOF fracture

A

full length femoral radiographs

495
Q

Surgical options of NOF#

A

Extracapsular - Dynamic Hip Screw (DHS)

Intracapsular - partial or total hip replacement based on pts age and mobility

496
Q

Complications of NOF repair

A

dislocation - more in total than hemi
perinprosthetic fracture
infection
mortality

497
Q

RF of hip OA

A
female
age
obesity
vit D deficiency 
trauma hx
anatomical abnormalities 
high impact sort participation
498
Q

gait abnormalities in hip OA

A

antalgic gait

End stage OA - trendelenberg gait

499
Q

Ddx of hip OA

A

trochanteric bursitis
sciatica
femoral neck fracture
gluteus medius tendonopathy

500
Q

how long can a hip replacement last for on average

A

10-15 years

501
Q

what is the normal oxygen extraction rate

A

250ml/min

502
Q

what is the apnoea timeth

A

time it takes to consume all oxygen in lungs when apnoeic. time you have to establish oxygen supply to pt

503
Q

Calculate apnoea time

A

(FRCxFraction of oxygen in alveolus) / O2 consumption

O2 consumption = 250ml/min
fraction of O2 in alveolus=0.14
Pts FRC varies so example 2500ml

(2500x0.14)/250

504
Q

How does pre oxygenation affect hypoxia apnoea time

A

Increases it by quite a few minutes

fraction of O2 in alveoli becomes 0.9

505
Q

what position should patients be in to be intubated

A

sniffing position

506
Q

why is the point of oxygen mask when pt is going under

A

to build up oxygen reserve in the lungs to make a longer hypoxia apnoea time

507
Q

where does the spinal cord end

A

L1/2

508
Q

what layers do you have to get through to get to the epidural space

A
skin
subcutaneous fat
supraspinous ligament
interspinous ligament
ligamentum flavum
epidural space
509
Q

at what level do you insert needle for spinal procedures

A

L4/5 (upper border of iliac crests)

510
Q

what space does a spinal go into

A

Subarachnoid

511
Q

how do you know you are in the subarachnoid space

A

CSF will leak out

512
Q

how do you know you are in the epidural space

A

loss of resistance to saline technique

513
Q

advantage of epidural over spinal

A

bc catheter is inserted it acts more long term than spinal does

514
Q

otherdifferences between spinal and epidural

A

spinal - done below L2, CSF flow confirms, narrow needle, 2-4ml drug volume, rapid onset

epidural - cervical to caudal, lasts for days, no CSF, wider needle, 10-20mls needed, slower onset

515
Q

indication for using neuroaxial blocks

A

surgery below level of umbilicus

516
Q

benefits of neuroaxial blocks opposed to GA

A

avoids complications of GA, lower risk of thrombosis, bleeding and in elderly theres a lower cognitive decline

517
Q

contraindications of neuroaxial blocks

A
infection at site
pt refusal
uncorrected hypovolaemia
allergies
increased ICP - can cause coning
518
Q

complications of neuroaxial blocks

A

minor - n+v, hypotension, hearing issues, shivering, itching, retention
Moderate - failure, postdural puncture headache, transient nerve injury
Major - infection (meningitis), cauda equina, haematoma, total spinal anaesthesia, permenant nerve injury/paralysis, cardiovascular collapse/death

519
Q

triad of anaesthetics

A

analgesia.
hypnosis
muscle relaxation

520
Q

4 stages of GA

A

Induction
Maintenance
Emergence
Recovery

521
Q

agents used for induction

A

IV propofol
Thiopentone sodium
Ketamine
Volatile agents eg sevoflurane

522
Q

paralytic agents used in GA

A

depolarising muscle relaxant - succinylcholine

Non depolarising muscle relaxant - atracurium, rocuronium

523
Q

indications for endotracheal intubation vs supraglottal airways such as iGel

A

ET - emergencies, major surgery, long duration surgery, full stomach

S - elective, well fasted, short duration

524
Q

maintenance drugs

A

TIVA eg propofol
fluid mx
abx, insulin etc
drugs for pain

525
Q

Side effects of GA

A
PONV
urinary retention
anaphylaxis
malignant hyperthermia 
drowsiness
paralytic ileus
aspiration
526
Q

what is local aneasthetic often used with and when is this pairing contraindicated

A

adrenaline as can increase the amount of local anaesthetic that can be used.
Cant use together in areas with an end arterial supply such as the digits, the oinna, penis or nose as vasoconstriction can cause gangrene

527
Q

max dose of lidocaine, lignocaine and bupivacains

A

lido - max dose 4.5mg/kg, with vasoconstriction 6-7mg/kg
Ligno 3mg/kg, with vasoconstriction 6mg/kg
Bup - 2mg/kg, with vasoconstriction 2.5mg/kg

528
Q

PE treatment in pregnant lady

A

dalteparin

529
Q

acute v chronic. graft vs host disease,

A

acute happens <100 days

tends to be more specifically symptomatic

530
Q

first line treatment of atelectasis + other mx

A

chest physio
+
oxygen, deep breathing exercises, PAIN CONTROL
CPAP if very severe

531
Q

which anaesthetic agent has anti emetic properties

A

propofol

532
Q

pathophys of RA

A

inflammatory proliferation of synovium forming a panus. Causes a thickened synovial membrane causing underlying articular damage

533
Q

Xray findings in RA

A
LESS
Loss of joint space
Erosions (periarticular)
Subluxation
Soft tissue swelling
534
Q

antibodies in RA

A

Rheumatoid factor and anti CCP

535
Q

Mx of Ra

A

acute - steroid injections
DMARDS eg methotrexate. Can combine if not working. Or change to biologics such a s anti tnf
NSAIDS

536
Q

extra articular features of RA

A

CAPS
3C- carpal tunnel, cervical myelopathy
3A - normocytic anaemia of chronic disease, macrocytic anaemia bc of folate deficiency, arteritis, amyloidosis
3P - pericarditis, pleural disease, pulmonary disease
3S - sjrogens, splenomegaly (Feltys), scleritis

537
Q

adverse effects of methotrexate

A
macrocytic anaemia
nausea
bone marrow suppression
hair thinning
hepatitis, cirrhosis, pneumonitis
538
Q

bloods to monitor in methotrexate

A

fbc - neutropaenia, anaemia
u+e’s
lft’s as can cause cirrhosis
folate

539
Q

side effect of carbimazole

A

agranulocytosis in first couple of months. May present with sore throat symptoms

540
Q

risk factors of GCA

A
older - 90% in over 60s
HLAB27
strong association with polymyalgia rheumatica 
women
white
541
Q

signs and symptoms of GCA

A
tender and pain over temple 
headache 
vision loss - sudden curtain comes down
jaw claudication
fever
malaise
joint pain - polymyalgia rheumatica
542
Q

why may biopsy not be successful in diagnosis. GCA

A

skip lesions

543
Q

mx of GCA

A

60-100mg po prednisolone for 2 weeks
if visual symptoms 1mg methylprednisolone IV for 1-3 days
low dose aspirin therapy for VT prophylaxis

544
Q

complication of GCA if not treated

permenant vision loss

A
545
Q

pattern. of pain in pmr

A

shoulders, hips, neck

546
Q

what condition is pmr associated with

A

GCA

547
Q

when to suspect pmr

A

elderly pt with new sudden onset of proximal limb pain and stiffness. Difficulty rising from chair or combing hair

548
Q

how is diagnosis of pmr madee

A

ESR or PV plus CRP

Temporal artery biopsy if symptoms of GCA

549
Q

treatment of pmr

A

15mg of pred daily for 1-2 yrs

should see dramatic response

550
Q

what gene are the sondhloarthropathies associated with

A

HLAB27

551
Q

presentation of ankylosing spondylitis

A

usually in young men

bilateral buttock pain, chest wall and thoracic pain

552
Q

examination findings in ankylosing spondylitis

A

loss of lumbar lordosis

Schobers test - mark skin 10cm above and 5 cm below PSIS, bend forward with straight legs, >20 cm increase is norma,

553
Q

ix of Ankylosing spondylitis

A

MRI spine and sacroiliac jointx, (more sensitive than xray)

554
Q

mx of as

A

NSAIDs
Physio
TNFinhibitors

555
Q

arthritis pattern in psoriatic arthritis

A

can be symmetrical or monon

556
Q

mx of psoriatic arthritis affecting spine

A

NSAIDS
DMARDS
TNF inhibitors

557
Q

reactive arthritis cause

A

sterile synovitis developing after distant infection eg salmonella, shigella, chlamydia

558
Q

presentation of reactive arthritis

A

few days - 2 weeks after infection, acute asymmetrical lower limb arthritis
may also have conjunctivitis and skin issues

559
Q

who is enteropathic arthritis common in

A

IBD

560
Q

why wouldnt you give NSAIDs in enteropathic arthritis

A

can flare IBD

561
Q

mx of enteropathic arthritis

A

DMARDS

562
Q

extra articular manifestations of AS

A
“The A Disease”
Anterior uveitis
AV block
Apical lung fibrosis
Amyloidosis
Aortic incompetence
563
Q

Features of inflammatory back pain acronym

A
IPAIN
insidious onset
Pain at night
Age <40
improvement with exercise
no inmprovement with rest
563
Q

Features of inflammatory back pain acronym

A
IPAIN
insidious onset
Pain at night
Age <40
improvement with exercise
no inmprovement with rest
564
Q

Sumboeyes spondyloarthropathy pneumonic

A
SPINE ACHE
Sausage fingers
Psoriasis
Iritis
NSAID Response
Enthesitis
Arthritis
Crohns
HLAB27
Elevated CRP, ESR, PV
565
Q

SLE Symptom Acronym

A
SOAP BRAIN
Serositis - pleurisy, pericarditis
Oral ulcers
Arthritis
Photosensitivity - malar rash
Blood disorders - low WCC, lymphopenia
Renal involvement - glomerulonephritis
Autoantibodies (ANA positive)
Immunologic tests eg low complements
Neurological disorder - seizures or psychosis
566
Q

Ix for SLE

A
ESR or PV
anaemia or leukopenia are common 
ANA positive 
urinalysis for renal disease
skin biopsy and renal biopsy can be diagnostic
567
Q

treatment of SLE

A

sun protection
healthy lifestyle advice in view of cardiovascular risk
hydroxychloroquine for rash and arthralgia
mycophenolate mofetil, azathioprine and rituximab
short courses of pred for flares

568
Q

complication of pernicious anaemia

A

gastric cancer

569
Q

what drug can cause gynaecomastia

A

spiro

570
Q

mx of BPH

A

Lifestyle - reduce caffeine, double voiding, bladder training
Medical - Tamsulosin (alpha blocker), Finasteride (5Alpha reductease inhibitor stops T-DHT)
Surgical - TURP

571
Q

Side effect of tamsulosin

A

lowers BP

572
Q

complications of TURP

A

retrograde ejaculation
incontinence
ED
Haemorrhage infection

573
Q

which parts of prostate do BPH and prostate cancer effect

A

BPH - transitional

Prostate - peripheral

574
Q

Ix for prostate cancer

A

DRE - hard irregular
multiparametric MRI
Biopsy
Bone scan

575
Q

when do renal stones need urgent intervention

A

uncontrollable pain
renal impairment
infection

576
Q

Mx of renal stones

A

Wait until pass - xray at 1 week to check
ESWL - ultrasound shock waves
PCNL - percutaneous nephrolithotomy

577
Q

lifestyle advice to help reduce recurrence of stones

A

increase citrate and fluid intake

reduce animal protein, salt and sugar

578
Q

gold standard renal stone ix

A

non contrast CT KUB

579
Q

size of kidney stone that requires active removal

A

> 10 mm

580
Q

what type of cancer in renal cell carcinoma

A

adenocarcinoma

581
Q

Triad of presentation of RCC

A

haematuria
loin pain
loin mass

582
Q

Potential finding of RCC in males

A

varicocoele

583
Q

difference in age of presentation between RCC and TCC

A

R - 55

T - 50-80

584
Q

RF of TCC

A

smoking

cyclophosphamide

585
Q

presentation of TCC

A

painless haematuria
frequency
urgency
obstruction

586
Q

type of cancer in bladder cancer

A

transitional cell

587
Q

presentation of bladder cancer

A

painless haematuria
recurrent UTIs
LUTS
Retention

588
Q

ix of bladder cancer

A

urine,
cystoscopy with biopsy
CT CAP to stage

589
Q

mx of bladder cancer

A
surveillance
intravesicular chemo with mitomycin C or BCG
Radical cystectomy with ileal conduit 
Palliation 
Long term catheter
590
Q

mx of acute urinary retention

A
analgesia
catheterise 
abx cover 
tamsulosin if caused by BOH
TWOC after 24-72hrs
591
Q

two types of chronic urinary retention

A

high pressure - bladder outflow obstruction. tend to get b/l hydronephrosis and decreased renal function
low pressure - no hydropnephrosis and normal renal function

592
Q

mx of high pressure chronic retention

A

catheterise

consider TURP

593
Q

low pressure urinary retention mx

A

avoid catheters as infection risk

Early TURP

594
Q

CI of suprapubic catheterisation

A

knnown or suspected bladder carcinoma
undiagnosed haematuria
previous lower abdo surgery

595
Q

complication of relieving urinary obstruction

A

post obstructive diuresis

need to keep hydrated to avoid fluid loss

596
Q

grading for prostate cancer

A

gleason

sum of 2 worst areas. 4+3 worse than 3+4

597
Q

exam of testicular tumour

A

painless lump - solid mass inseperable from testis

598
Q

diagnostic test of testicular cancer

A

scrotum US

599
Q

stress incontinence mx

A

pelvic floor exercises
ring pessart
artificial urinary sphincter

600
Q

urge incontinence mx

A

bladder training, weight loss
Anti AChM - tolterodine
oxybutynin - can cause dry eyes and dry mouth
surgical - botox (CI in myasthaenia gravis)
sacral nerve stimulation (S3)
Clam ileocystoplasty

601
Q

COPD Stepwise mx nice guidelines

A

SABA/SAMA
No asthmatic features or features suggesting steroid responsiveness
- add LABA + LAMA
- if taking a SAMA discontinue and switch to a SABA

Asthmatic features/features suggesting steroid responsiveness
- LABA + ICS
if still breathless triple therapy - LABA + ICS + LAMA

602
Q

when to give amoxicillin to an under 2 with acute otitis media

A

bilateral infection in children under 2, otorrhoea, perforated tympanic membrane and symptoms not improving after 3 days.

603
Q

when to operate on AAA

A

if greater than 5.5cm or greater than 1cm growth in a year

604
Q

Modified Glasgow Criteria. What does score mean

A
PaO2 <7.9
Age >55
Neutrophils >15x10^3
Calcium <2
Renal function Urea >16
Enzymes LDL >600
Albumin <32
Sugar >10

Score greater than 3 indicates severe pancreatitis with high risk of progressing to SIRS. Need to be treated in HDU or ITU

604
Q

Modified Glasgow Criteria. What does score mean

A
PaO2 <7.9
Age >55
Neutrophils >15x10^3
Calcium <2
Renal function Urea >16
Enzymes LDL >600
Albumin <32
Sugar >10

Score greater than 3 indicates severe pancreatitis with high risk of progressing to SIRS. Need to be treated in HDU or ITU

605
Q

complications of acute pancreatitis

A
sepsis
ARDS
SIRS
chronic pancreatitis
DM
606
Q

sit up right milestone

A

6-8 months

607
Q

stand up right holding object

A

9

608
Q

pulls to stand

A

10 months

609
Q

walks with assistance

A

12 months

610
Q

walk unaided

A

15 months

611
Q

cut off for walking - to think about developmental delay

A

18 months

612
Q

run uinaided

A

18 months

613
Q

what does 6 week check involve

A

Mum

  • stopped bleeding
  • wound sites
  • post partum depression

Child

  • weigh
  • general assessment of what they look like - rashes birth mark
  • head circumference
  • tone, reflexes - babinski, moro’s
  • exam everything - organs, genitalia, hips (ddh)
  • barlows for ddh
614
Q

5 childhood exanthems

A
1st disease Measles
2nd disease Scarlet fever 
3rd disease Rubella
5th disease erythema infectiosum - slap cheek
6th disease roseola infantum
615
Q

presentation of measles

A
4days of fever
3C’s - Cough, coryza, conjunctivitis
Koplik spots - pathognomonic. white palatal spots
rash starts behind ears 
confluent erythematous rash
616
Q

scarlet fever presentation

A
post strep infection
strawberry tongue
fever, phayngeal erythema
pasta lines in skin folds 
‘sandpaper’ rash
lymphadenopathy
617
Q

ix for strep

A

anti streptolysin O titre

618
Q

mx of scarlet fever

A

abx as strep

probs penicillin V or benpen

619
Q

rubella presentation

A

swollen lump behind ear
coughing, sneezing, aching, fever
red spots along palet - forschemer

620
Q

complication of rubella in pregnant lady

A

congenital rubella syndrome

621
Q

fifth disease presentation

A

lace like and reticular on trunk and limbs, blotchy on cheeks
fever, coryza, before the slapped cheek shows up
Not infective when slap cheek shows

Think Safeguarding!!!! non accidental injury

622
Q

cause of slapped cheek pa

A

parvovirus B19

623
Q

advice for all exanthems

A

stay away from preggers and immunocompromised

624
Q

roseola infantum (3 day fever)

A

mostly torso

3 days of high fever then suddenly drops to normal

625
Q

mongolian spots

A

big blue spots
THINK SAFEGUARDING
birth marks so last a long time

626
Q

mx of nappy rash

A

education for parents

E45, sudacrem

627
Q

chicken pox rash description

A

itchy generalised discrete erythematous vesicular rash with blisters
(varicella is vesicular)

628
Q

bronchiolitis cause

A

Respiritory Syncitial virus (rsv)

629
Q

bronchiolitis presentation

A
<2 yrs
recessions - sucking in diaphragm 
grunting/wheezing
nasal flaring
blue baby 
dry cough
doesnt want to play, not feeding
630
Q

rf for bronchiolitis

A

premature

Chronic lung disease eg CF

631
Q

mx of bronchiolitis

A
supportive
safety net!!!
reasurre
paractetamol, ibuprofen
admit babies that are severe
631
Q

mx of bronchiolitis

A
supportive
safety net!!!
reasurre
paractetamol, ibuprofen
admit babies that are severe
632
Q

type of wheeze in asthma and why

A

polyphonic - multiple airways

633
Q

what could cause a monophonic wheeze

A

foreign body, one airway effected

634
Q

when can asthma not be diagnosed in children

A

<5yrs

635
Q

Asthma BTS guidelines

A
1 SABA as required
2 low dose ICS
3 LABA + low dose ICS
4 increase dose of ICS or LTRA 
5 refer for specialist care
636
Q

colic definition

A
637
Q

advice for colic for parents

A
638
Q

dx to think of when child wetting bed

A

type 1 dm

abuse, trauma

639
Q

what is perthes disease

A

avascular necrosis of femoral head in children

640
Q

GCS categories and point

A
Eyes Motor Verbal
Eyes 
1 no eye opening
2 eye open to pain
3 eye open to verbal command
4 eye open spontaneously
Motor
1 no motor response
2 extension response to pain
3 flexion response to pain
4 withdraws from pain
5 localises pain
6 obeys command
Verbal
1 no verbal response
2 incomprehensible sounds
3 inappropriate words
4 confused
5 orientated
641
Q

at what gcs do you need to consider intubation

A

8

642
Q

what effect of chlorpheramine sets it apart from other anti histamines

A

sedative

643
Q

how quick do you need to donall sepsis 6 interventions

A

within an hr

644
Q

types of shock

A
SHOCK
Sepsis/anaphlaxis
Hypovolaemia
Obstructive eg tamponade
Cardiogenic
K(cortisol) - adrenal
645
Q

cause of hypocalcaemia in pancreatitis

A

fat necrosis of pancreas

breakdown products bind to calcium reducing levels

646
Q

adverse signs in arrythmias

A

shock
chest pain
heart failure
syncope

647
Q

adenosine dosing for SVT

A

6,12,18

648
Q

what size pneumothorax do you need to insert needles

A

2 cm

649
Q

pneumothorax algorithm

A
650
Q

why stop metformin in aki

A

bc of lactic acidosis risk

651
Q

ddx for chronic limb ischaemia

A

spinal stenosis (claudication)

652
Q

ix for chronic limb ischaemia

A

lipid profile
ABPI
blood glucose
duplex ultrasound

653
Q

benefits of exercise in claudicationc

A

encourages formation of collateral vessels

654
Q

calculating ABPI

A

ankle systolic pressure/brachial systolic pressure

655
Q

normal ABPI value and abnormal values

A

0.9-1.4 normal
<0.9 peripheral artery disease
>1.4 calcification of arteries

656
Q

mx of chronic arterial insufficiency

A

lifestyle - weight loss, optimal diabetic control, encourage exercise
pharm - clopidogrel 75mg, statin
surgery - bypass or percutaneous transluminal angioplasty

657
Q

test used for acute limb ischaemia and description

A

beurgers test - raise leg to 45 degrees

positive = feet become pale, not the angle. And then out foot doen below bed

658
Q

pain pattern in acute limb ischaemia

A

worse at night

hang leg out of bed to relieve

659
Q

definitive mx of acute limb ischaemia

A

catheter embolectomy

660
Q

complications associated with post revascularisation

A

o2 radicals leading to inflammation and oedema

be aware of compartment syndrome and muscular necrosis

661
Q

how long for complete acute limb ischaemia to cause extensive tissue necrosis

A

6hrs

662
Q

important cause of varicose veins to remember and ask about

A

DVT

663
Q

mx of varicose veins

A

reassurance
compression
stripping

664
Q

complications of varicose veins

A

haemorrhage
symptoms and signs of chronic venous insufficiency eg lipodermatosclerosis, thrombophlebitis, venous ulcers and eczema, haemosiderin staining

665
Q

mx of chronic venous insufficiency

A

compression socks
analgesia
if ulcer then full compressive treatment

665
Q

mx of chronic venous insufficiency

A

compression socks
analgesia
if ulcer then full compressive treatment
venous stunting if very severe

666
Q

causes of aaa

A
degenerative eg smokers
connective tissue disorders eg marfans 
congenital 
familial
infective
dissection
667
Q

numbers to know for aaa

A

if <5.5 surveillance - every 3 months if 4.5 to 5.5. Every year if 3-4.4cm
if grows >1cm in 12 months, surgery
if >5.5 surgery
aneurysm = 1 and a half times normal size
normal size = approx 2cm

668
Q

surgery for aaa

A

endovascular stent

669
Q

screening for aaa

A

men aged 65

670
Q

ix for vascular problems

A
ABPI
Duplex uss
doppler
lipid profiles
clotting screens
671
Q

arterial vs neuropathic ulcers

A

arterial
- punched out, unhealthy wound bed, lateral malleolus, pressure points, necrotic tissue

neuro
- painless, plantar aspect, surrounded by callous

672
Q

what value must ABPI be for safe compression bandaging

A

> 0.8

673
Q

what fractures have a high association with vascular injury

A

supracondylar humeral - brachial
high tibial - popliteal
posterior dislocation of shoulder - axillary

674
Q

what does a biphasic sound on doppler indicate? what is normal sound?

A

artery stiffening

triphasic

675
Q

most likely diagnosis of breast lump by age

A

young - fibroadenoma
50s - cysts
old - cancer

676
Q

gp mx of ALL breast lumps

A

2ww

677
Q

duct ectasia - waht is it

A

dilatation of lactiferous duct

678
Q

presentation of duct ectasia

A
mastalgia
yellow discharge from nipples
can feel like a lump
peri menopausal
nipple retraction bc of inflammation
679
Q

mx of duct ectasia

A

analgesia and abx

680
Q

what can present similarly to duct ectasia

A

peri ductal mastitis

681
Q

presentation of breast cyst

A

tender smooth lump, well defined

usually multiple

682
Q

mx of breast cyst

A

self resolving - analgesia

if large, aspirate and send off for cytology if blood or if lump doesn’t go away

683
Q

what is involved in breast triple assessment

A

mammogram
exam
biopsy

684
Q

presentation of intraductal papilloma

A

40-50
subaereolar region (less than 1 cm away from nipple)
can mimic carcinoma on imaging so requires biopsy usually
clear or bloody discharge from nipple

685
Q

benign breast lumps

A
fibroadenoma
lipoma
intraductal papilloma
duct ectasia
breast cyst
686
Q

what is cyclical mastalgia

A

pre menopausal women

pain and increased nodularity of breast related to hormonal changes throughout menstrual cycle

687
Q

presentation of DCIS

A

cheesy discharge
confined to ducts
lump
asymptomatic

688
Q

mx of DCIS

A

wide local excision or mastectomy

689
Q

what is pagets disease of the nipple

A

erythematous, ulceration
itchy, flaky
painful
involves both areola and nipple

some have underlying malignancy so need to biopsy

690
Q

main ddx for pagets and how do you differentiate this from pagets

A

eczema - spares nipple

691
Q

mx of pagets

A

excision of nipple and areola or mastectomy

radiotherapy if underlying malignancy

692
Q

what is most common breast cancer

A

invasive ductal carcinoma

693
Q

signs of breast cancer

A

nipple retraction/inversion
nipple discharge - bloody
hard lump in breast +/- axilla
skin changes - peau de orange, pagets disease of the nipple

694
Q

rf of breast cancer

A
fhx 
smoking
increased age 
nulliparity 
first child when >30
early menarche and late menopause
radiation exposure
hx of breastfeeding
695
Q

first line treatment of mastitis in breastfeeding ladies and why

A

continue breastfeeding or expressing milk throughout treatment
can give fluclox for 10-14 days

696
Q

complication of mastitis

A

breast abcess

697
Q

ages targeted in breast screening

A

50-70

698
Q

how often are women screened for breast cancer

A

every 3 years

698
Q

how often are women screened for breast cancer

A

every 3 years

699
Q

what to do with oral diabetic medications the day before surgery

A

should be taken as normal

699
Q

what to do with oral diabetic medications the day before surgery

A

should be taken as normal

700
Q

mx of ascending cholangitis

A

iv abx

ERCP after 24-48hrs to relieve obstruction

701
Q

likely presentation of overactive bladder in men and mx drug

A

voiding and storage symptoms on background of BPH

add anti muscarinic such as tolterodine or oxybutynin

702
Q

first line medical treatment for fissures

A

stool softeners
topical diltiazem
to relax sphincter and facilitate healing

703
Q

main ix for a young male with acute prostatitis

A

sti screen

704
Q

what type of drug is propofol

A

GABA receptor agonist

705
Q

what anaesthetic agents can cause malignant hyperthermias

A

suxamethonium, volatile liquid anaesthetics eg sevoflurane, isoflurane

706
Q

when is dalteparin started perioperatively

A

about 6hrs POST op

707
Q

what does % mean in drugs eg 2% lidocaine

A

eg 2g of lidocaine in 100ml

708
Q

timetable and 5Ws of causes of post operative fever

A
Day 1-2 Wind - pneumonia, atelectasis, PE
Day 3-5 Water - UTI
Day 5-7 wound - infection or abscess
Day 5+ Walking - DVT or PE
Anytime Wonder about drugs
709
Q

before ileus becomes symtpomatic with nausea and vomiting, what can it cause

A

hypovolaemia, electrolyte disturbances

710
Q

what is a potential serious cause of AF after a gastrointestinal surgery

A

anastomotic leak

711
Q

what can local anaesthetic toxicity be treated with

A

IV 20% lipid emulsion

712
Q

examples of quinolones

A

ciprofloxacin

levofloxacin

713
Q

adverse effects of quinolones eg ciprofloxacin

A

lowers seizure threshold
tendon damage/rupture
increased QT interval

714
Q

most common organisms causing acute cholecystitis

A

E coli
klebsiella
strep

715
Q

what is reynolds pentad for

A

ascending cholangitis

Charcots + signs of shock

716
Q

FeverPAIN

A
Fever in past 24hrs
Purulent tonsils
Attended within 3 days
Inflammation severe
No cough and coryza
717
Q

syndrome caused by giving aspirin to under 16

A

Reye’s syndrome - cerebral oedema

718
Q

CKD Staging

A
eGFR.                 ACR
G1 >90.             A1 <3
G2 60-89.         A2 3-30
G3a 45-59.       A3 >30
G3b 30-44
G4 15-29
G5 <15
719
Q

talk through peritoneal dialysis process

A

Dialysate fluid pumped into peritoneum and left for few hrs
Dialysate has high glucose content so high osmolality. Draws water and electrolytes out into the peritoneum
fluid is then drained

719
Q

talk through peritoneal dialysis process

A

Dialysate fluid pumped into peritoneum and left for few hrs
Dialysate has high glucose content so high osmolality. Draws water and electrolytes out into the peritoneum
fluid is then drained

720
Q

complications of peritoneal dialysis

A

encapsulated peritoneal sclerosis
peritonitis
hernias

721
Q

haemodialysis process

A

fistula
blood passed through dialyser.
Dialyser has lots of semi permeable channels surrounding by a constant flow of dialysate (going opposite direction to blood)
3x a week for 4 hrs

722
Q

if pt cant have fistula for haemodialysis, what can you do instead

A

insert a permcath

723
Q

rank rrt in order of effectiveness

A

renal transplant
haemodialysis
peritoneal dialysis

724
Q

complications of haemodialysis

A

infection/bacteraemia
reactions to dialysers
cramps
anaemia

725
Q

what criteria make you unsuitable for RRT (no additional survival benefit)

A

age >80 with WHO performance score of 3 or more

726
Q

what can you do for pts who are unsuitable for RRT

A

active conservative mx - treating symptoms, palliative care

726
Q

what can you do for pts who are unsuitable for RRT

A

active conservative mx - treating symptoms, palliative care

727
Q

contraindications of renal transplant

A
cardiac failure
active infection or malignancy 
reversible renal disease
non compliance to treatment 
short life expectancy
substance abuse
727
Q

contraindications of renal transplant

A
cardiac failure
active infection or malignancy 
reversible renal disease
non compliance to treatment 
short life expectancy
substance abuse
728
Q

3 types of kidney transplant

A

living donor related
living donor unrelated
dead donor

728
Q

3 types of kidney transplant

A

living donor related
living donor unrelated
dead donor

729
Q

induction therapy for kidney transplant

A

immunosupression drugs such as methylprednisolone in combo with basiliximab

730
Q

maintenance drugs after renal transplant

A

steroids
calcineurin inhibitors such as tacrolimus and cyclosporin
mycophenalate mofitil
azathioprine

731
Q

complications of renal transplant

A

rejection
infection bc of immunosupression so opportunistic
cancers - 3 times more likely to develop any cancer so need monitoring
NODAT - new onset diabetes after transplant

732
Q

colic definition

A

infant cries for more than 3 hrs a day for more than 3 days a week

733
Q

pathophys of DKA

A

insulin deficiency causes decreased glucose uptake
Glycogenolysis. Lipolysis and muscle breakdown for gluconeogenesis which causes hyperglycaemia with ketones as a by product
Acidosis
Osmotic diuresis - loss of water and electrolytes

733
Q

pathophys of DKA

A

insulin deficiency causes decreased glucose uptake
Glycogenolysis. Lipolysis and muscle breakdown for gluconeogenesis which causes hyperglycaemia with ketones as a by product
Acidosis
Osmotic diuresis - loss of water and electrolytes

734
Q

symptoms of hypoglycaemia

A
Sweating
Tremor
Anxious
Dizzy
Drowsy
Seizures
Loss of consciousness 
blurred vision
palpitations
735
Q

what value is hypoglycaemic

A

<3.9

736
Q

non diabetic causes of hypoglycaemia

A

Addisons
Exogenous drugs
insulinoma
too mich exercise not enough food

736
Q

non diabetic causes of hypoglycaemia

A

Addisons
Exogenous drugs
insulinoma
too mich exercise not enough food

737
Q

signs of DKA

A
Dry mucus membranes
Sunken eyes
Tachycardia
Hypotension
Ketotic breath
Kussmaul resp.
Altered mental state
Hypothermia
738
Q

Symptoms of DKA

A
Polyuria
polydipsia
nausea/vomiting 
blurred vision
abdo pain - bc acidosis can cause ileus
739
Q

diagnostic criteria for dka

A

ketonuria
hyperglycaemia
acidosis

740
Q

ketone level in dka

A

> 3

741
Q

why do HHS not have ketonuria

A

bc still have some insulin sensitivity

741
Q

why do HHS not have ketonuria

A

bc still have some insulin sensitivity which can shpress lipolysis enough to prevent ketone production

742
Q

complications of HHS and prevention

A

DVT, stroke bc VeRy DEHYDRATED

VTE tprophylaxis

743
Q

causes of HHS

A

diabetic meds non compliance
infection
MI
bowel infarct

744
Q

glucose level in hhs

A

usually >30

745
Q

signs of hhs

A

signs of severe dehydration

  • dry mucus membranes
  • sunken eyes
746
Q

osmolality in hhs

A

> 320

747
Q

mx of hhs

A

IV fluids
Potassium
IV insulin
DVT prophlaxis - bc increased risk bc of dehydration

748
Q

meds used in basal bolus dosing

A

long acting - lantus

rapid acting - novorapid

749
Q

advise to give diabetics starting on insulin

A

how to monitor blood glucose
injection technique - rotate sites to avoid lipohypertrophy
risk of DKA and hypoglycaemia and how to recognise
inform DVLA
complications of poor glycaemic control

750
Q

diagnosis of diabetes

A

fasting glucose >7 on 2 occasions or 1 w/ symptoms
Random glucose >11.1 on 2 occassions or 1 w/ symptoms
HbA1c (type 2 only) >48

751
Q

when do you not use HbA1c

A
pregnant
child
type 1 
renal failure
HIV
steroid use
752
Q

Contraindications of metformin

A

eGFR<30

alcohol intoxication

753
Q

gliclazide (sulfonylurea) ADRs

A

weight gain

hypoglycaemia

754
Q

glitazones ADRs

A

hypoglycaemia
fluid retention
bladder cancer risk
increased fracture risk

755
Q

CI of glitazones

A

heart failure

756
Q

DPP4 (sitagliptin) CI and why

A

hx of pancreatitis as small risk pf pancreatitis

756
Q

DPP4 (sitagliptin) CI and why

A

hx of pancreatitis as small risk pf pancreatitis

757
Q

presentation of raynauds

A

young female
vasospasm of digits causing colour changes in response to cold or stress stimulus
white-blue-red

758
Q

Raynauds developing over age of 30 should alert you to what diseases:

A

scleroderma
SLE
dermato and polymyositis
sjogrens

drug induced: beta blockers

759
Q

mx of raynauds

A

keep warm
avoid smoking
ccb are first line eg nifedipine
then phosphodiesterse 5 inhibitors

760
Q

complications of raynauds

A

digital ulcers
infection
gangrene

761
Q

ix for raynauds

A

nail fold capillaroscopy

762
Q

Examples of small vessel vasculitis

A

Granulomatosis with polyangiitis (wegeners)
IgA vasculitis (henoch schonlein purpura)
microscopic polyangiitis

763
Q

example of medium vessel vasculitis

A

kawasaki disease

764
Q

example of large vessel vasculitis

A

GCA

765
Q

causes of secondary vasculitis

A

infectiom
drugs
malignancy
connective tissue disease

much more common than primary vasculitis

765
Q

causes of secondary vasculitis

A

infectiom
drugs
malignancy
connective tissue disease

much more common than primary vasculitis

766
Q

symptoms of vasculitis

A

general - fever, loss of appetitie. weight loss, fatigue

specific - depends on vessels affected eg could get haematuria, haemoptysis, neuropathy,visual loss

767
Q

treatment of vasculitis

A

1st line - corticosteroids

2 - cytotoxic meds or biologic agents eg methotrexate, azathioprine. rituximab

768
Q

what is dermatomyositis

A

inflammation of striated muscle

769
Q

presentation of dermatomyositis

A

insidious onset of muscle proximal weakness, often painless
May have SOB or rash
Raynauds commonly associated

770
Q

Ix in dermatomyositis

A

raised muscle ALT but normal liverALT
ANA - positive
Anti Jo 1
MRI - myositis well demonstrated

771
Q

mx of dermatomyositis

A

high dose corticosteroids
long term control with azathioprine
Sun protection importqnat - hydroxychloroquine useful

772
Q

why is there a risk of aspiration. pneumonia in dermatomyositis

A

bc oesophagus is striated muscle so swallowing may be affected

772
Q

why is there a risk of aspiration. pneumonia in dermatomyositis

A

bc oesophagus is striated muscle so swallowing may be affected

773
Q

signs of dermatomyositis

A

photosensitive rash - scalp face and neck
Gottrons papules - linear plaques on dorsum of hand
dilated nail fold capillaires and dry cracked palms
periorbital oedema
heliotrope rash - violet rash of eyelids

774
Q

pathophys of systemic sclerosis

A

multisystem autoimmune disease
increased fibroblast activity resulting in abnormal growth of connective tissue
vascular damage and fibrosis

775
Q

2 stypes of systemis sclerosis

A

limited and diffuse

776
Q

symptoms and signs of limited scleroderma

A
CREST
Calcinosis
Raynauds
E oesophageal dysmotility
Sclerodactyly
Telangiectasia
777
Q

ix of systemic sclerosis

A
xray hands - calcinosis
CXR for pulmonary disease
ECG, ECHO
ANA +
anti centromere ab with limited scleroderma
778
Q

mx of scleroderma

A
no cure 
ccb for raynauds
methotrexate
ACEi prevent HTN crisis 
prednisolone for flares
779
Q

complication of diffuse SSc

A

scleroderma renal crisis causing HTN and renal failure

779
Q

complication of diffuse SSc

A

scleroderma renal crisis causing HTN and renal failure

780
Q

mnemonic for symptoms and signs of sjogrens

A
MAD FRED
Myalgia
Arthralgia
Dry mouth
Fatigue
Raynauds
Enlarged parotids
Dry eyes
781
Q

Ix for sjogrens

A

salivary gland biopsy
Anti Ro and Anti La
RF and Anti - ds DNA

782
Q

antibodies associated with SLE

A

anti Ro anti La
Anti dsDNA
antiphospholipid ab

783
Q

Mx of sjogrens

A

treat symptoms eg avoid dry atmospheres, eye drops, skin emollients, artificial saliva

784
Q

what do you see on biobsy in sjogrens

A

focal lymphocytic infiltration of exocrine glands

785
Q

symptoms and signs of hypermobility

A
pain around joints
fatigue
marfanoid habitus, 
arachnodactyly
drooping eyelids, myopia
hernia and prolapses
786
Q

Mx of hypermobility

A

strengthening exercises to reduce subluxation/dislocation

paracetamol for pain

787
Q

what score is used for hypermobility

A

beighton score - max score of 9.
hands on floor
elbow backwards, knee backwards. thumb to wrist, 90 degree pinky

788
Q

pathophys of OA

A

progressive degeneration of articular cartilage accompanied by new bone formation and capsular fibrosis

789
Q

rf of OA

A

obesity
joint malalignement through trauma or muscle weakness
female

790
Q

mx of OA

A
physio
weight loss
NSAIDS
intra articular steroid injections
joint replacements
791
Q

nodal OA sign names

A

bouchards and heberdens nodes

792
Q

main 3 symptoms pf fibromyalgia

A

pain everywhere
no energy
unrefreshed sleep

792
Q

main 3 symptoms pf fibromyalgia

A

pain everywhere
no energy
unrefreshed sleep

793
Q

mx of fibromyalgia

A

dealing with depression, fatigue, sleep disturbance
if drugs then low dose amitrypytline or pregabalin may be effective
CBT

794
Q

Rf of osteoporosis

A
Age, female, fhx, hx of low trauma fracture
Low BMI
premature menopause
calcium/vit d deficiency
inadequate ohysical activity
smoking
excessive alcohol intake 
steroids
795
Q

diagnosis of osteoporosis

A

DEXA scan
T score x number of SDs from mean bone density of same gender at age of peak density (25)
T score - 2.5 or less = osteoporosis

796
Q

scores in osteoporosis

A

T score

Z score

797
Q

mx of osteoporosis

A

risk modification - weight bearing exercise, vit D3 supplements, reduce alcohol and smoking, dietary advice regarding calcium

calcium and vit D supplements plus:
1st line oral bisphosphonates
2nd line denosumab

798
Q

pathophys of gout

A

hyperuricaemia

deposition of monosodium urate crystals in joints and soft tissues

799
Q

rf for gout

A
age >40
male
increased purine uptake (fish)
high fructose
obese
chf
renal disease
HtN
smoking
DM
Meds eg thiazides
800
Q

mx of gout

A

risk modification - reduce weight,diet mod, reduce alcohol

NSAIDS, steroids, colchicine for flares
Allopurinol for chronic. don’t stop allopurinol in acute flares if already on it but dont start if not

801
Q

when is colchicine CI

A

pregnancy
renal failure - eGFR <10
blood disorders

802
Q

appearance of gout in joint aspirate

A

negatively birefringent needle shaped crystals

803
Q

who is pseudogout common in

A

older women with OA

804
Q

what abx can be used as prophylaxis for infective exacerbations of COPD

A

azithromycin

805
Q

which CCB is most likely to precipitate pulmonary oedema in a pt with known chronic heart failure

A

verapamil

806
Q

why do pts with coeliacs need a pneumococcal vaccine

A

bc of hyposplenism

807
Q

what skin condition can co amox cause

A

erythema multiforme

808
Q

examples of UKMEC4

A
current breast cancer
migraine with aura - bc of ischaemic stroke risk
BMI >40
smokers >15 a day
liver tumours
809
Q

CI for IUD

A

PID

Fibroids causing distortion of uterus

809
Q

CI for IUD

A

PID

Fibroids causing distortion of uterus

810
Q

causes of avn of femoral head

A

long term steroid use
trauma chemo
alcohol excess

810
Q

causes of avn of femoral head

A

long term steroid use
trauma chemo
alcohol excess

811
Q

what is an important back pain differential to consider in IVDU

A

psoas abscess

812
Q

what disease is most associated with carpal tunnel

A

rheumatoid

813
Q

what can ivdu with underlying infective endocarditis present with?

A

discitis

814
Q

position of leg in posterior hip dislocation

A

shortened and internally rotated

814
Q

position of leg in posterior hip dislocation

A

shortened and internally rotated

814
Q

position of leg in posterior hip dislocation

A

shortened and internally rotated

815
Q

what does pain on radial styloid of wrist indicated

A

de quervains tenosynovitis

815
Q

what does pain on radial styloid of wrist indicated

A

de quervains tenosynovitis

816
Q

if a diabetic is ill what advice should you give them about their insulin and why

A

continue to take it and frequently check blood glucose. At increased risk of DKA

816
Q

if a diabetic is ill what advice should you give them about their insulin and why

A

continue to take it and frequently check blood glucose. At increased risk of DKA so stopping would be very dangerous

817
Q

what does curb65 tell us and how does this change mx plans

A

risk of death

if 3/4, NICE recommend urgent admission to hospital

818
Q

which diabetic drug is CI in HF

A

glitazones bc cause fluid retention

819
Q

which HTN drug class should be avoided in preggers

A

ACEi

820
Q

drugs to avoid in asthma

A

NSAIDS
B Blockers
Adenosine

821
Q

what drugs are CI in heart failure

A

pioglitazone
verapamil
NSAIDs with caution
Class 1 antiarrhythmics eg flecainide

822
Q

which cancer does tamoxifen increase risk of

A

endometrial

822
Q

which cancer does tamoxifen increase risk of

A

endometrial

823
Q

what drug can precipitate digoxin toxicity

A

thiazides bc they can cause hypokalaemia

824
Q

what is the most important prognostic factor in paracetamol overdose

A

pH

825
Q

what anti emetics do you not give in bowel obstruction and why

A

pro kinetics eg metoclopramide and domperidone

Can increase risk of perforation

825
Q

what anti emetics do you not give in bowel obstruction and why

A

pro kinetics eg metoclopramide and domperidone

Can increase risk of perforation

826
Q

causes of raised ALP

A

Biliary obstruction eg - gall stones, hepatic tumour (SOL)

Bone eg - pagets, osteomalacia, bone mets,

827
Q

interpreting ALT and ALP

A

ALT 10 fold increase with ALP less than 3 fold increase = hepatocellular injury
ALP 3 fold increase with ALT less than 10 fold increase = cholestasis

828
Q

ALT/AST ratio meanings

A

ALT>AST = chronic liver disease

ALT

829
Q

what is purpose of gamma GT

A

determines whether rise in ALP is hepatobilliary or non hepatobilliary eg vit d deficiency, pagets, osteomalacia

830
Q

when catheterising pt for urinary retention how can you tell whether it is acute or chronic based off the fluid output

A

Chronic >1.5 L and is often painless

831
Q

histological features of crohns

A

granuloma formation
lymphocytes infiltration
transmural inflammation

832
Q

which IBD has perianal disease and what does this mean

A

Crohns - perianal abcesses, fissures, fistulae

833
Q

complications of crohns

A
perianal abscess or fistulae, 
perforated bowel
small bowel obstruction
colonic carcinoma
malnutrition
833
Q

complications of crohns

A
perianal abscess or fistulae, 
perforated bowel
small bowel obstruction
colonic carcinoma
malnutrition
834
Q

Treatment of Wilsons

A

penicillamine - heavy metal antagonist

835
Q

Psoas abscess presentation

A

lower abdo pain, relived by hip flexion, worsens with extension and internal rotation
low grade fever

835
Q

Psoas abscess presentation

A

lower abdo pain, relived by hip flexion, worsens with extension and internal rotation
low grade fever

836
Q

what cancers is COCP protective against

A

ovarian and endometrial

837
Q

reversal agent for dabigatran

A

Idarucizumab

838
Q

reversal agent for doacs other than dabigatran

A

adexanate alfa

839
Q

ullipristal acetate CI

A

asthma

840
Q

why does resp rate increase in sepsis

A

to compensate for metabolic acidosis, blow of CO2

841
Q

mx of mechanical back pain

A
physio
rest
education : manual handling technique
analgesia: paracetamol+-NSAIDS+-Codeine
muscle relaxant - diazepam (short term)
842
Q

what is a disc prolapse pathophys

A

herniation of nucleus polposus through annulus fibrosus

843
Q

what discs are most likely to prolapse and which nerves are most commonly affected

A

L4/L5 and L5/S1

L5 root. S1 root

844
Q

Presentation of an L4/5 prolapse

A

Compression of L5 so:
Sensory loss along lateral aspect of leg and dorsum of foot
Weakness or abscence of great toe extension
lower back pain
limited spinal flexion and extension
pain on straight leg raise

845
Q

Presentation of L5/S1 prolapse

A

general:
back pain
reduced spine flexion and extension
pain on passive straight leg lift - lasegue sign

specific:
S1 compression
Sensory loss on sole of foot
Motor weakness of plantar flexion and eversion

846
Q

big syndrome to rule out in a central herniation

A

cauda equina

846
Q

big syndrome to rule out in a central herniation

A

cauda equina

846
Q

big syndrome to rule out in a central herniation

A

cauda equina

847
Q

imaging of spine to rule out cauda equina

A

MRI

848
Q

what is spondylolisthesis

A

displacement of a vertebra usually anteriorly to the one below

849
Q

what is spinal stenosis

A

facet joint arthritis causing narrowing of spinal canal

850
Q

presentation of spinal stenosis

A

CLAUDICATION - aching or heavy buttock or lower limb pain when walking
Pain relieved by flexion
Pain worsened by extension

851
Q

Mx of spinal stenosis

A

NSAIDS
Epidural steroid injection
Canal decompression surgery

852
Q

surgery for cauda equina

A

decompression - laminectomy or discectomy

853
Q

causes of cauda equina

A
disc prolapse
malignancy 
infection
haemorrhage in spinal canal
spinal stenosis
853
Q

causes of cauda equina

A
disc prolapse
malignancy 
infection
haemorrhage in spinal canal
spinal stenosis
854
Q

cauda equina red flags

A
perianal numbness
fecal incontinence
painless urinary retention
bilateral sciatica
erectile dysfunction
Anal sphincter laxaty
855
Q

imaging choices for achilles rupture

A

US or MRI

but most diagnosed on clinical examination alone

856
Q

tumour marker in hepatocellular carcinoma

A

serum AFP

857
Q

Mx of pericarditis

A

NSAIDS and colchicine

858
Q

what is electrical alternans and what can cause it

A

pericarditis causing exudative effusion causes heart to change position every beat
seen by alternating sizes of qrs complexes

859
Q

what signs would you see with electrical alternans

A

Beck’s triad - basically cardiac tamponade

860
Q

what does a posterior MI look like on ECG

A

st depression from leads V1-3 and tall R waves

861
Q

why not give oxygen if >94 in MI?

A

Risk of reperfusion injury

862
Q

3 common causes of ascites

A

cirrhosis
Chronic cardiac failure
cancer - mets most common

863
Q

what ix should be performed on all pts with ascites and why

A

diagnostic ascitic tap (cell count and MC&S)

for spontaneous bacterial peritonitis

864
Q

mx of ascites

A

spironolactone

pericentesis if tense - if pt is in a lot of discomfort and pain

865
Q

what prophylactic treatments can reduce risk of GI bleeding from varices

A

propanalol (non selective beta blocker)

endoscopic band ligation

866
Q

MSK complication of cirrhosis and how do you screen for it

A

osteoporosis - DEXA

867
Q

most important predictor of bleeding in surgery

A

bleeding history NOT Coag screen

868
Q

what dies joint aspirate in RA look like

A

yellow fluid
high lymphocyte count
polymorphonuclear neutrophil predominance

869
Q

threshold for blood transfusion in normal people vs people with ACS

A

Hb <70g/L in normal

<80g/L in those with ACS

870
Q

what rash is mycoplasma pneumoniae associated with

A

erythema multiforme - target shaped rash

871
Q

what 4 changes happen in hypovolaemic shock

A

decreased CO
Increased HR
Reduced left ventricular filling pressures
Reduced BP

872
Q

what must pts do for 6 weeks before getting tested for coeliacs

A

eat gluten

872
Q

what must pts do for 6 weeks before getting tested for coeliacs

A

eat gluten

872
Q

what must pts do for 6 weeks before getting tested for coeliacs

A

eat gluten

873
Q

xray findings in as

A

subchondral erosisons
sclerosis
squaring of lumbar verterbre
sacroilitis

874
Q

what drug should be discontinued 48hrs following a contrast CT

A

metformin bc of renal impairment risk

875
Q

what is the medication used for hepatic encephalopathy

A

lactulose

875
Q

what is the medication used for hepatic encephalopathy

A

lactulose

876
Q

ADRs of beta blockers

A

tired, dizzy, lightheaded - sx of slow HR
hypotension
cold peripheries
bronchospasm if have asthma

877
Q

how is pseudogout different to gout

A

deposition of calcium pyrophosphate in the joints rather than monosodium urate
typically affects knee first
Associated with hypothyroidism, hypoparathyroid and hypomagnasaemia.
positively bifringent rhomboid shaped crystals

878
Q

indications for a permanent pacemaker

A
heart block - 2:2 and 3
symptomatic brady
sick sinus
heart failure 
drug resistant tachy
879
Q

what drugs to stop in heart block

A

beta blockers, CCBs

880
Q

pathophys of Dresslers

A

immune response to the pericardium post MI

happens few weeks after MI

881
Q

complication of nephrotic syndrome

A
renal vein thrombosis
infection
hyperlipidaemia
hypocalcaemia
acute renal failure
882
Q

differentiate hf swelling from nephrotic syndrome swelling

A

nephrotic you get periorbital as well as leg

883
Q

what condition causes muddy brown casts

A

rhabdo causing Acute Tubular Necrosis

884
Q

what increases risk of rhabdo when prescribed with statins

A

clarithromycin

885
Q

electrolyte complications of rhabdo

A

metabolic acidosis
hyperkalaemia
hypocalcaemia

885
Q

electrolyte complications of rhabdo

A

metabolic acidosis
hyperkalaemia
Hyperphosphataemia
hypocalcaemia

886
Q

mx of rhabdo

A

treat hyperkalaemia
IV fluid
can also give IV sodium bicarbonate or dialysis if severe renal failure

886
Q

mx of rhabdo

A

treat hyperkalaemia
IV fluid
can also give IV sodium bicarbonate or dialysis if severe renal failure

887
Q

what drug is used to prevent progression of renal failure in diabetic nephropathy and MOA

A

ACEi to treat microalbuminuria - dilates efferent arteriole which reduces glomerular capillary filtration pressure and GFR and reduces risk of glomerulosclerosis

888
Q

what supplements can reduce absorption of levothyroxine and cause hypothyroidism relapse

A

iron and calcium

should be taken 4 hrs apart from levothyroxine

888
Q

what supplements can reduce absorption of levothyroxine and cause hypothyroidism relapse

A

iron and calcium

should be taken 4 hrs apart from levothyroxine

888
Q

what supplements can reduce absorption of levothyroxine and cause hypothyroidism relapse

A

iron and calcium

should be taken 4 hrs apart from levothyroxine

888
Q

what supplements can reduce absorption of levothyroxine and cause hypothyroidism relapse

A

iron and calcium

should be taken 4 hrs apart from levothyroxine

888
Q

what supplements can reduce absorption of levothyroxine and cause hypothyroidism relapse

A

iron and calcium

should be taken 4 hrs apart from levothyroxine

889
Q

resuktnof OGTT in acromegaly

A

no suppression of growth hormone after oral glucose bolus

890
Q

management for phaeochromocytoma

A

phenoxybenzamine (irreversible alpha blocker) otherwise risk hypertensive crisis

For at least 3 weeks then surgery

890
Q

management for phaeochromocytoma

A

phenoxybenzamine (irreversible alpha blocker) otherwise risk hypertensive crisis

For at least 3 weeks then surgery

891
Q

what incontinence drug should be avoided in frail older people

A

oxytocin as increased risk of confusion and falls

891
Q

what incontinence drug should be avoided in frail older people

A

oxytocin as increased risk of confusion and falls

891
Q

what incontinence drug should be avoided in frail older people

A

oxytocin as increased risk of confusion and falls

891
Q

what incontinence drug should be avoided in frail older people

A

oxytocin as increased risk of confusion and falls

891
Q

what incontinence drug should be avoided in frail older people

A

oxytocin as increased risk of confusion and falls

892
Q

what incontinence drug should be avoided in frail older people

A

oxytocin as increased risk of confusion and falls

893
Q

first line pharm mx of delirium

A

haloperidol

But if have parkinsons or lewy body then use lorazepam

894
Q

factor differentiating delirium from dementia

A

ACUTE onset
impairment of conscious level in dementia
fluctuation of symptoms - worse at night
abnormal perceptions - delusions and hallucinations
agitation

895
Q

cause of secondary polycythaemia in copd

A

response to chronic hypoxia

896
Q

abx for acute IE of COPD

A

co amox for 5 days

or doxy if pen allergic

897
Q

what rash is pathognomonic of coeliacs

A

dermatitis herpetiformis - itchy vesicular extensor rash

897
Q

what rash is pathognomonic of coeliacs

A

dermatitis herpetiformis - itchy vesicular extensor rash

897
Q

what rash is pathognomonic of coeliacs

A

dermatitis herpetiformis - itchy vesicular extensor rash

898
Q

what pathogen is associated with secondary pneumonia after a preceding influenza infection

A

staph aureus

899
Q

virus associated with eczema herpeticum

A

herpes simplex 1 and 2

900
Q

who does a kelbsiella pneumonia mostly effect

A

alcoholics

900
Q

who does a kelbsiella pneumonia mostly effect

A

alcoholics

901
Q

what rash is associated with mycoplasma pneumoniae

A

erythema multiforme

902
Q

lfts in biliary colic

A

normal

903
Q

what blood can differentiate between upper and lower GI bleed

A

urea - high in upper

903
Q

what blood can differentiate between upper and lower GI bleed

A

urea - high in upper

904
Q

what surgical procedure is used for upper rectum

A

anterior resection

904
Q

what surgical procedure is used for upper rectum

A

anterior resection

904
Q

what surgical procedure is used for upper rectum

A

anterior resection

905
Q

most common heart murmur in IVDU

A

tricuspid regurge

906
Q

bisphosphonate example

A

allendronic acid

906
Q

bisphosphonate example

A

allendronic acid

907
Q

scoring system for RA

A

DAS28

908
Q

mx of excema (different severity levels)

A

mild

  • emollients eg E45
  • topical hydrocortisone 1%

moderate

  • emollients
  • betnovate for 48hrs until flare controlled
  • topical hydrocortisone 1% for face
  • dry bandages
  • antihistamines - certirizine, loratidine

Severe

  • emollients
  • betnovate or eumovate for 5 days
  • dry bandaging
  • sedative antihistamine if struggling to sleep
  • avoid triggers
908
Q

mx of excema (different severity levels)

A

mild

  • emollients eg E45
  • topical hydrocortisone 1%

moderate

  • emollients
  • betnovate for 48hrs until flare controlled
  • topical hydrocortisone 1% for face
  • dry bandages
  • antihistamines - certirizine, loratidine

Severe

  • emollients
  • betnovate or eumovate for 5 days
  • dry bandaging
  • sedative antihistamine if struggling to sleep
  • avoid triggers
908
Q

mx of excema (different severity levels)

A

mild

  • emollients eg E45
  • topical hydrocortisone 1%

moderate

  • emollients
  • betnovate for 48hrs until flare controlled
  • topical hydrocortisone 1% for face
  • dry bandages
  • antihistamines - certirizine, loratidine

Severe

  • emollients
  • betnovate or eumovate for 5 days
  • dry bandaging
  • sedative antihistamine if struggling to sleep
  • avoid triggers
909
Q

advice for topical steroids

A
1 finger tip unit
downward motion in direction of heair
dont rub in
wash hands thotoughly afterwards
wait 30 mins or more before applying emollient
909
Q

advice for topical steroids

A
1 finger tip unit
downward motion in direction of heair
dont rub in
wash hands thotoughly afterwards
wait 30 mins or more before applying emollient
909
Q

advice for topical steroids

A
1 finger tip unit
downward motion in direction of heair
dont rub in
wash hands thotoughly afterwards
wait 30 mins or more before applying emollient
910
Q

side effects of topicla steroids

A
skin thinning
temporary stunging/burning
stretch marks
acne rash
mild lightening of the skin
911
Q

diagnosis to consider if eczema becomes weepy, bleeding, blisters

A

eczema herpeticum

912
Q

acne management

A

non pharm

  • frangrance free cleaners
  • reduce make up use
  • do not squeeze
  • healthy diet

Mild

  • topical retinoids eg benzoyl peroxide, adapalene
  • topical abx (erythromycin) in combination with topical retinoids

Moderate
- oral abx (lymecycline or doxycycline) with topical retinoid

Severe
- roacutain

913
Q

psoriasis mx

A
  • avoid precipitating factors eg beta blockers
  • topical corticosteroids
  • emollients
  • vit D analogues
  • coal tar preps
  • calcinuerininhibitors - tacrolimus
914
Q

dexamethasone supression

A

low dose

  • low cortisol = normal
  • high cortisol = cushings syndrome

high dose

  • low cortisol = cushings disease
  • high cortisol, low ACTH = adrenal cushings
  • high cortisol, high ACTH = ectopic cushings
915
Q

causes of secondary hyperaldosteronism

A

heart failure
renin secreting tumour
cor pulmonale
cirrhosis

915
Q

causes of secondary hyperaldosteronism

A

heart failure
renin secreting tumour
cor pulmonale
cirrhosis
Renalartery stenosis bc reduced perfusion

915
Q

causes of secondary hyperaldosteronism

A

heart failure
renin secreting tumour
cor pulmonale
cirrhosis
Renal artery stenosis

916
Q

renin:aldosterone

A

low r:low a = cushings
low r:high a = primary hyperaldosteronism
high r:low a = addisons
high r:high a = secondary hyperaldosteronism

917
Q

tumour markers to know
CEA,AFP, Ca199, Ca125, Ca153

A
CEA -  colorectal
AFP - hepatocellular
Ca199 pancreatic
Ca125 ovarian
Ca153 breast
918
Q

coeliac disease histology

A

villous atrophy, crypt hyperplasia, lymphocyte infiltration

919
Q

lvh causes

A

htn
aortic stenosis
hypertrophic cardiomyopathy

920
Q

ix for htn

A

24hr BP
USS kidney
MRI renal/aorta
bloods

921
Q

complications of HTN

A

aortic dissection
strokes
renal failure

921
Q

complications of HTN

A

aortic dissection
strokes
renal failure

922
Q

hypertrophic cardiomyopathy - where is it thick

A

thickened septum

923
Q

causes of hf

A
ischaemic heart disease
valvular heart disease
arryhtmias
alcohol and drugs
cancer drugs 
congenital heart disease
924
Q

what med is efficaciis in controlling future symptoms of svt

A

flecainide

924
Q

what med is efficaciis in controlling future symptoms of svt

A

flecainide

924
Q

what med is efficaciis in controlling future symptoms of svt

A

flecainide

924
Q

what med is efficaciis in controlling future symptoms of svt

A

flecainide

925
Q

why leg swelling in h f

A

reduced RV function so reduced venous return

blood pools in veins in legs and so increased pressure forces fluid out of the vasculature

926
Q

risk factors of rcc

A
smoking
male
Caucasian
dialysis 
obesity 
aromatic hydrocarbons 
htn
927
Q

?rxof a renal tumour????

A

partial or radical nephrectomy
cryotherapy
percutaneous radio frequency ablation

928
Q

what cancer treatment does not work on renal tumours

A

chemo

928
Q

what cancer treatment does not work on renal tumours

A

chemo

929
Q

What is the most common type of thyroid cancer?

A

Papillary cancer

930
Q

Digoxin monitoring

A

No routine monitoring required, unless toxicity is suspected.

If toxicity suspected, measure 8-12 hours after last dose.

931
Q

Red flags warranting further ix for ovarian cysts

A

Irregular solid tumour
Ascites
At least 4 papillary structures
Irregular multilocular
Very strong blood flow

932
Q

Rx of pre menstrual syndrome

A

Lifestyle - smoking, alcohol
COCP
SSRI if severe

933
Q

Most common thyroid cancer
Prognosis
Most likely complication

A

Papillary
Good
Spread to cervical lymph nodes

934
Q

Features of digoxin toxicity

A

generally unwell, lethargy, nausea & vomiting, anorexia, confusion, yellow-green vision
arrhythmias (e.g. AV block, bradycardia)
gynaecomastia

935
Q

Mx digoxin toxicity

A

Digibind
correct arrhythmias
monitor potassium

936
Q
A
937
Q

Presentation of myasthenia gravis crisis

A

Bilateral ptosis
Slurred speech
Respiratory failure

938
Q

Triple therapy for H Pylori

A

PPI
Amoxicillin/metronidazole
Clarithromycin

939
Q

What is pseudomembranous colitis What abx are associated with it
Blood results (WCC)

A

Inflammation of the colon associated with an overgrowth of C Diff. Get abdo pain, diarrhoea and fever
Leukocytosis

Clindamycin
Penicillins
Cephalosporins

940
Q

Drugs causing diarrhoea

A

Abx
PPI
NSAIDs
Digoxin

941
Q

What is Klinefelters syndrome?
- karyotype?

A

47XXY
Hypergonadotrophic hypogonadism
- tall stature
- low volume testes
- sparse pubic hair
- gynaecomastia

High LH and FSH, low testosterone

942
Q

What drug can be used for rapid sequence induction?
- intubation

A

Suxamethonium

943
Q

Clozapine has been missed for 2 doses.
What are the next steps with regards to this?

A

If clozapine has been missed for 48 hours, should retitrate doses slowly.
- after a break of 48 hours or more, side effects are worse

944
Q

List some ototoxic medications

A

Aminoglycosides- gentamicin, neomycin

Furosemide- esp when give by rapid IV infusion. Usually reversible

Cisplatin

Aspirin- can cause tinnitus

945
Q

In what cases should you send an MSU if UTI is suspected?

A

-Aged over 65
-Visible or non visible haematuria
-Pregnancy
- Men

946
Q

Medical management ectopic pregnancy?
What is important to make the patient aware of? (think future)

A

IM methotrexate

The methotrexate will be teratogenic for 3 months- ensure adequate contraception

947
Q

A patient has an ectopic source of ACTH.
What will a high dose dexamethasone suppression test show?

A

Cortisol not suppressed

ACTH not suppressed

948
Q

What drug should be considered in patients with T1DM with BMI over 25?

A

Metformin

949
Q

What screening test would you use for diabetic neuropathy of feet?

A

10 g monofilament

950
Q

Causes of pseudo-Cushings?

A

Alcohol excess
Severe depression

951
Q

First line ix for acromegaly?

A

Serum IGF-1

Then OGTT

952
Q

1st line treatments for diabetic neuropathy?

A

Pregabalin
Gabapentin
Duloxetine
Amitriptyline

Any of the above

953
Q

Drug causes of gynaecomastia?

A

-spironolactone (most common drug cause)
-cimetidine
-digoxin
-cannabis
-finasteride
-GnRH agonists e.g. goserelin, buserelin
-oestrogens, anabolic steroids

954
Q

What result would you expect to see on FBC following 3 weeks of high dose steroids?

A

Neutrophilia- although steroids are immunosuppressive, steroids have contradictory effect on neutrophils